You are on page 1of 57

Gabungan soal

1. Mrs. S, 29 YO, G3P1A1 39 weeks GA, referred by a midwife with prolonged second stage.
Physical examination revealed normal vital sign. Obstetric examination revealed contraction was
3x/10’/35” and FHR was 160 bpm. Estimated fetal weight was 3100 gr with previous baby was 3000
gr. Vaginal examination: fully dilated, no amniotic membrane, lowest part of the fetus was head with
descent of the head in Hodge IV. Denominator was minor fontanelle at the left anterior.

What methods will you choose to terminate this condition?


A. Spontaneous delivery
B. Augmentation
C. Embryotomy
D. Forceps extraction
E. Caesarean Section

2. Mrs. S, 29 YO, G3P1A1 39 weeks GA, referred by a midwife with prolonged second stage.
Physical examination revealed normal vital sign. Obstetric examination revealed contraction was
3x/10’/35” and FHR was 160 bpm. Estimated fetal weight was 3100 gr with previous baby was 3000
gr. Vaginal examination: fully dilated, no amniotic membrane, lowest part of the fetus was head with
descent of the head in Hodge IV. Denominator was minor fontanelle at the left anterior.

What are the indication for assisted vaginal delivery?


A. Uterine rupture
B. PPROM
C. Postpartum hemorrhage
D. Fetal anomaly
E. Fetal or maternal distress

3. A 33-year-old G0P0 woman comes to your office for her initial prenatal visit. She tested positive
with two home pregnancy tests and ultrasound revealed 6-7 wga pregnancy. As her pregnancy
continues, you would expect her cardiac output to increase by which of the following mechanisms:
A. First an increase in stroke volume, then an increase in heart rate.
B. A decrease in systemic vascular resistance.
C. Cardiac output would not change significantly until the third trimester.
D. An increase in systemic vascular resistance facilitated by elevated progesterone level.
E. An increase in heart rate alone.

4. You see a patient in your office who had a thyroidectomy from Graves Disease. She is now
hypothyroid on thyroid hormone replacement. You monitor her TSH and keep it between 0.5 and 2.5.
You increase her thyroid hormone replacement each trimester and her TSH remains around 2.0 during
the entire pregnancy.

In addition to monitoring her TSH, what other additional testing should you perform during her
pregnancy?
A. Amniocentesis to determine if the fetus is affected by Graves Disease.
B. A detailed fetal ultrasound at 18 to 20 weeks and again in the third trimester should be
performed given the increased risk of fetal goiter.
C. Fetal echocardiogram to evaluate for cardiac abnormalities.
D. Umbilical Dopplers to monitor for placental dysfunction.
E. MCA Dopplers to monitor for fetal anemia.

5. You see a patient in your office who had a thyroidectomy from Graves Disease. She is now
hypothyroid on thyroid hormone replacement. You monitor her TSH and keep it between 0.5 and 2.5.
You increase her thyroid hormone replacement each trimester and her TSH remains around 2.0 during
the entire pregnancy.

Management of hypothyroidism in pregnancy is:


A. Women after thyroidectomy may require higher doses of levothyroxine.
B. TSH levels measured weekly.
C. Thyroxine dose is adjusted by 100 mcg increments until TSH values become normal.
D. Increased thyroxine requirements begin as early as 12 weeks.
E. None of the above.

6. A 35-year-old P3 with a Pap smear showing high-grade squamous intraepithelial lesion of the
cervix (CIN III) has an inadequate colposcopy. Cone biopsy shows squamous cell cancer that has
invaded only 1 mm beyond the basement membrane. There are no confluent tongues of tumor, and
there is no evidence of lymphatic or vascular invasion. The margins of the cone biopsy specimen
are free of disease. How would you classify or stage this patient’s disease?
a. Carcinoma of low malignant potential
b. Microinvasive cancer
c. Atypical squamous cells of undetermined significance
d. Carcinoma in situ
e. Invasive cancer, stage Ia

7. A 35-year-old P3 with a Pap smear showing high-grade squamous intraepithelial lesion of the
cervix (CIN III) has an inadequate colposcopy. Cone biopsy shows squamous cell cancer that has
invaded only 1 mm beyond the basement membrane. There are no confluent tongues of tumor, and
there is no evidence of lymphatic or vascular invasion. The margins of the cone biopsy specimen
are free of disease. Of the following, appropriate therapy for this lesion is
a. External beam radiation
b. Implantation of radioactive cesium
c. Simple hysterectomy
d. Simple hysterectomy with pelvic lymphadenectomy
e. Radical hysterectomy

8. A 35-year-old P3 with a Pap smear showing high-grade squamous intraepithelial lesion of the
cervix (CIN III) has an inadequate colposcopy. Cone biopsy shows squamous cell cancer that has
invaded only 1 mm beyond the basement membrane. There are no confluent tongues of tumor, and
there is no evidence of lymphatic or vascular invasion. The margins of the cone biopsy specimen
are free of disease. In CIN 3, which of the following statement is correct?
a. Neoplastic cells extend into the proximal cervical glands whereas the basal membrane remains
intact
b. Cytoplasmic maturation in the upper third of mucosa
c. There is extension of neoplastic epithelium into endocervical glands and the connective tissue
d. Neoplastic cells derive from both squamous and glandular endocervical epithelium
e. Glandular acinar formations extend into the endocervical stroma

9. A 34-yo woman came to clinic to have her 6 weeks postpartum check. She just gave birth to a
3200 grams-baby delivered vaginally 3 months ago. She had a complain of unable to hold liquid
stool and flatus. On examination the external genitalia looks normal. What is the next diagnostic
plan for this patient?
a. Defecography
b. Ultrasound of the anal sphincter
c. Anal manometry
d. Pelvic CT scan
e. Electromyography of the anal sphincter.

10. A 34-yo woman came to clinic to have her 6 weeks postpartum check. She just gave birth to a 3200
grams-baby delivered vaginally 3 months ago. She had a complain of unable to hold liquid stool
and flatus. On examination the external genitalia looks normal. What is the next management of
this case?
a. Observation on the symptoms and reevaluation 3 months
b. Give constipating agent to prevent liquid stool
c. High fiber diet, hydration of minimum 2 L and reevaluation 6 weeks
d. Sacral neuromodulation
e. Reconstructive surgery

11. A 34-yo woman came to clinic to have her 6 weeks postpartum check. She just gave birth to a 3200
grams-baby delivered vaginally 3 months ago. She had a complain of unable to hold liquid stool
and flatus. On examination the external genitalia looks normal. If we want to do a reconstructive
surgery to this patient, what is the recommended suture material that should be use?
a. Chromic catgut
b. Polyethylene teraphthalate
c. Polypropylene
d. Poliglecaprone
e. Polyglactin

12. A 24 year old female came to emergency department with right lower quadrant abdominal pain
since 6 hours ago. She has no fever. Her last menstrual period was 2 weeks ago. Her cycles was
28 days. On physical examination found vital sign within normal limit, right lower quadrant
abdominal tenderness without sign of peritonitis, and on ultrasound found uterus and left ovary
normal, seen mass on right ovary sized 3,5cm, with thin wall and spider web sign, no free fluid
found. Pregnancy test was negative. Hb level 12,3g/dL.
What is the most likely diagnosis of this patient?:

a. Dermoid cyst
b. Torsion of the ovarian cyst
c. Cyst rupture
d. Hemorrhagic of the corpus luteum
e. Tubo ovarial abscess

13. A 24 year old female came to emergency department with right lower quadrant abdominal pain
since 6 hours ago. She has no fever. Her last menstrual period was 2 weeks ago. Her cycles was
28 days. On physical examination found vital sign within normal limit, right lower quadrant
abdominal tenderness without sign of peritonitis, and on ultrasound found uterus and left ovary
normal, seen mass on right ovary sized 3,5cm, with thin wall and spider web sign, no free fluid
found. Pregnancy test was negative. Hb level 12,3g/dL.

What is the next management plan for this patient?


a. Conservative management
b. Antibiotic intravenous for 5 days
c. Emergency Laparoscopic cystectomy
d. Emergency Laparotomy
e. Puncture of the Douglass pouch

14. A 46-year-old P2 obese woman is referred from her primary-care physician because of increasingly
heavy and painful menses over the last 18 months. She has tried an oral contraceptive with some
improvement of her bleeding but no improvement in her pain. She has never had an abnormal Pap
smear and states she has never had any infections, “down there.” Her only medical problems are
her obesity and hypertension. On examination, you note normal external genitalia, vagina, and
cervix. However, her uterus is slightly enlarged, mildly tender, and softer than you expected. She
has no adnexal mass or tenderness.
Which imaging study listed below would BEST differentiate between adenomyosis and uterine
fibroids?

a. Pelvic ultrasound

b. Pelvic CT

c. Sonohysterogram

d. Pelvic MRI

e. Hysterosalpingogram

15. A 46-year-old P2 obese woman is referred from her primary-care physician because of increasingly
heavy and painful menses over the last 18 months. She has tried an oral contraceptive with some
improvement of her bleeding but no improvement in her pain. She has never had an abnormal Pap
smear and states she has never had any infections, “down there.” Her only medical problems are
her obesity and hypertension. On examination, you note normal external genitalia, vagina, and
cervix. However, her uterus is slightly enlarged, mildly tender, and softer than you expected. She
has no adnexal mass or tenderness.
After further evaluation suggesting adenomyosis, your patient wants to proceed with hysterectomy
because she is tired of bleeding and experiencing pain. You explain to her that she needs to undergo
a test prior to scheduling her hysterectomy. What test does the patient need to undergo?

a. Colposcopy

b. Mammogram

c. Endometrial biopsy

d. Colonoscopy

e. Chest X-ray

16. An 8-year-old girl is brought to your office by her parents concerned that she has started developing
breasts too soon. Physical examination reveals a Tanner stage II breast and pubic hair growth.
You reassure the parents that this is normal. What is typical pubertal sequence?

a. Menarche, pubarche, thelarche, accelerated growth

b. Pubarche, thelarche, accelerated growth, menarche

c. Thelarche, pubarche, accelerated growth, menarche

d. Accelerated growth, thelarche, pubarche, menarche

e. Pubarche, accelerated growth, thelarche, menarche


17. An 8-year-old girl is brought to your office by her parents concerned that she has started developing
breasts too soon. Physical examination reveals a Tanner stage II breast and pubic hair growth.
When can the patient expect her first menses?

a. At 16 years of age

b. At 14 years of age

c. At 11 years of age

d. At 9 years of age

e. It is impossible to tell from the data provided

18. A 51-year-old woman presents to your office amenorrhea for the past year. Since she has not had
any hot flashes, she is wondering if she is menopausal. Which of the following is NOT a classic
symptom of menopause?

a. Hot flashes

b. Insomnia

c. Visual changes

d. Vaginal atrophy

e. Night sweats

19. A 20-year-old young woman presents to your office with a complaint of amenorrhea. She notes
that she has never had a menstrual period, but that she has mild cyclic abdominal bloating. She
reports normal breast development starting at age 12. She reports that she has become sexually
active, but she finds intercourse painful. Her past medical and surgical history is unremarkable. On
physical examination, you note normal appearing axillary and pubic hair. Her breast development
is normal. Pelvic examination reveals normal appearing external genitalia, and a shortened vagina
ending in a blind pouch. Which of the following tests would be your first step in determining the
diagnosis?

a. Karyotype

b. Pelvic ultrasound

c. Serum follicle-stimulating hormone (FSH)

d. Diagnostic laparoscopy

e. Serum estradiol

20. An 18-year-old young woman presents to your office with a complaint of amenorrhea. She notes
that she has never had a menstrual period, but that she has mild cyclic abdominal bloating. She
reports normal breast development starting at age 12. She reports that she has become sexually
active, but she finds intercourse painful. Her past medical and surgical history is unremarkable. On
physical examination, you note normal appearing axillary and pubic hair. Her breast development
is normal. Pelvic examination reveals normal appearing external genitalia, and a shortened vagina
ending in a blind pouch. You perform a bedside ultrasound and find normal appearing bilateral
ovaries as well as an absent uterus and fallopian tubes. What is your most likely diagnosis?

a. Imperforate hymen

b. Transverse vaginal septum

c. Müllerian agenesis

d. Androgen insensitivity syndrome (AIS)

e. Late onset congenital adrenal hyperplasia

21. A 34 years old woman with primary infertility 3 years, oligomenorrhea and a body mass index
(BMI) of 26. Transvaginal ultrasound shows multiple small follicle size 5-8 mm in both ovaries.
HSG shows bilateral patent tubes. Her partner’s semen analysis shows a volume of 2.5 ml, pH of
7 and a sperm count of 25 million/ml. According to the current International Guidelines. Which
of the following is true regarding ovulation induction in PCOS?
a. Gonadotropin cannot be used as a first line treatment
b. Use of gonadotropin correlate with lower cost of ovulation induction
c. If there are two mature follicles ovulation trigger should be cancelled
d. Use of aromatase inhibitor reserved for Clomiphene failure patients
e. Clomiphene citrate has a higher risk of multiple pregnancy compared to aromatase inhibitor

22. A 30 years old woman came with chief complain of no menstruation for 6 months. She has a
history of 3 times surgery for endometriosis cyst. From ultrasound uterus size and shape within
normal limit endometrial line 3 mm, both ovaries size is smaller than normal without any
follicles present. Which of the following management is appropriate for this condition?
a. Adminiester E-P challenge test using combined oral contraception
b. Administer progestin challenge test
c. Measure Ca125 and CRP level
d. Measure FSH and estradiol level
e. None of the above

23. A 26 years old unmarried woman came with chief complain of abdominal mass since 5 months
ago. She also has a history of heavy menstrual bleeding with regular cycle since 6 months
accompanied with dysmenorrhea VAS 8. From examination the uterus enlarged globular with
limited mobility, no mass was palpable on both adnexas. Which of the following ultrasound
features that correlate with this pathology?
a. Lobulated mass in the myometrium with clear border
b. Circumferential vascularity around the mass in the uterus
c. Normal junctional zone
d. Fan shaped shadowing with ill-defined border
e. Thickened endometrial line

24. A 30 years old patient came with complaint of infertility. Her husband is a 33-year-old who has
had a semen analysis, which was reported as normal. On further history, the patient reports that
her periods have been quiet irregular over the last year and that she has not had period in the last
3 months. She also reports of weight gain and abnormal hair growth. From ultrasound we found
multiple follicles size 2-8 mm in both ovaries. Which of the following correlate to this condition?
A. Day 3 FSH level 20 IU
B. Serum AMH level 2,6 ng/ml
C. Positive Clomiphene citrate challenge test
D. Midluteal progesterone level of 4 ng/ml
E. Follicle antral basal count of 12

25. Mrs. N, 27 years old with chief complain of infertility for 2 years with history of severe
dysmenorrhea. From hysterosalpingography, both tubes were patent. Pelvic ultrasound found
bilateral cystic mass with internal echo sized 25 and 40 mm in diameter, her antral follicle count
was 12. Her husband sperm examination was within normal limit. What is the next appropriate
management?
A. Offer her IUI
B. Perform laparoscopy cystectomy and adhesiolysis
C. Give GnRH analog for 3 months continue with IUI
D. Give Dienogest 1x2mg for 6 months
E. Offer her IVF

26. A 50-year-old lady presents to gynecology outpatient clinic with postmenopausal bleeding. A
thorough clinical workup reveals an endometrial thickness of 7mm with an irregular, echogenic, thickly
septated ovarian mass on her left ovary. What type of tumor is responsible for this lady’s clinical
presentation?
A. Dysgerminoma
B. Embryonal carcinoma
C. Epithelial adenocarcinoma of the ovaries
D. Endodermal sinus tumor
E. Granulosa-theca cell tumor

27. A 28 years old patient, P0, presents to your clinic for fertility workup. She had been married
for 2 years with regular intercourse. Her menstrual cycle is normal. Her general status was normal.
Vaginal examination revealed normal findings. The following month she came back with the result of
hysterosalpingography (see the picture below). What will be your next step ?
A. Order Semen
Analysis
B. Schedule operative laparoscopy
C. Gives clomiphene citrate and plan for natural conception
D. Gives clomiphene citrate and plan for intrauterine insemination
E. Plan for IVF

28. A 32-year-old woman presents to your office. She complained about her sexual problems of
pain during sexual intercourse since giving spontaneous birth of her second child 6 months ago. She
feels a normal desire to engaged in sexual intercourse. She is now still breatsfeed the baby and she is in
combine oral contraceptive pills. What is the most common cause of this problem?
A. Infection
B. Episotomy scar
C. Combine Oral contraceptive pills
D. Fatigue
E. Breastfeeding

29. A 60-year-old P2 presents to the urogynecology clinic with complaints of urinary incontinence.
She has urinary urgency and can’t make it to the bathroom before leaking a large amount of urine. She
urinate 12 time during the day and gets up two to three times per night to urinate. What is the next
examination plan for this case?
A. Voiding diary
B. Urinalysis
C. Pelvic floor ultrasound
D. Pad test
E. Urodynamic study

30. Your patient is a 13-year-old adolescent girl who presents with cyclic pelvic pain. She
has never had a menstrual cycle. She is afebrile and her vital signs are stable. On physical
examination, she has age-appropriate breast and pubic hair development and normal external
genitalia. there is no bulge in the introitus of vagina and no hymenal structure seen. You obtain
a transabdominal ultrasound, which reveals a hematocolpos and hematometra. . What is the
most likely diagnosis?
A. Distal Transverse vaginal septum
B. Longitudinal vaginal septum
C. Imperforate hymen
D. Distal Vaginal agenesis
E. MRKH

31. A 89-year-old female patient with multiple, serious medical comorbidities presents to
discuss options for treatment of her high-grade prolapse. The prolapse is externalized and
becoming ulcerated from friction against her undergarments. Her main priority is to “fix or get
rid of this thing,” but her primary care provider has cautioned against a lengthy or open
abdominal procedure. She is not interested in future intercourse. What can you offer this
patient?
A. Total colpocleisis
B. Partial colpocleisis
C. Sacrospinous hysteropexy
D. Ring pessary.
E. Manchester Fothergyl operation

32. A 38 years old multi gravid woman complains of the painless loss of urine, beginning
immediately with coughing, laughing, lifting, or straining. cessation of the activity do not stops the
urine loss. She also complains of frequency with small-voided volume of urine, but no urgency. This
history is most suggestive of
A. Fistula
B. Stress incontinence
C. Urge incontinence
D. Overflow incontinence
E. Mixed incontinence

33. A 28-year-old woman is hoping to become pregnant soon. She is worried about her history of
acute pelvic inflammatory disease (PID) when in college 8 years ago. Which of the following is the
pathogenesis of infertility due to history of PID?
A. Direct destruction of tubal mucosa by chlamydia
B. Inflammatory response followed with fibrosis to protein released by chlamydia
C. Inflammation of the endometrium
D. Ovulatory dysfunction due to pelvic inflammatory disease
E. Inflammation that hinders spermatozoa movement inside the uterus

34. A 72 years old woman P6 came to outpatient clinic with chief complaint of bulging mass
protrudes from vagina since 3 months ago. The mass usually occurs during activity and also when she
defecate , and disappear when lying down. There were no difficulty in voiding and defecation. No
urinary leakage during coughing and sneezing. She is not sexually active. If on the Pelvic Organ
Prolapse Quantification examination result showing below, what is the diagnosis of this patient?

Aa Ba C

+3 +4 +1

GH Pb TVL

5 2 8 A. Uterine prolapse grade 2, cystocele grade 2, rectocele grade 1


B. Uterine prolapse grade 4, cystocele grade 3, rectocele grade 2
Ap Bp D C. Uterine prolapse grade 2, cystocele grade 3, rectocele grade 2
D. Uterine prolapse grade 3, cystocele grade 3, rectocele grade 2
-1 -1 -2
E. Uterine prolapse grade 3, cystocele grade 2, rectocele grade 2

35. A 45 years old woman presents to your office for


consultation regarding her symptoms of menopause. She stopped having periods 13 months ago after
BSO operation and is having severe hot flushes. The hot flushes are causing her considerable stress.
Which of the following medication that you will give for hormonal therapy?
A. Estrogen only therapy
B. Biphasic combined oral contraception
C. Monophasic combined oral contraception
D. Triphasic combined oral contraception
E. Sequential estrogen-progestin therapy

36. A patient present to you with pain and swelling in the vulva. On examination you find a
reddish bulge on the vaginal introitus at 4 o’clock positions sized 3 cm, cystic and pain on palpation.
What treatment of choice for this condition?
A. Antibiotic for 7 days
B. Incision and drainage
C. Marsupialization
D. Cystectomy
E. Excision of the glands
37. Mrs. N, 37 years old had just undergone laparoscopic procedures. Her chief complaint was
infertility for 6 years with history of severe dysmenorrhea. From hysterosalpingography, both tubes
were non-patent. Pelvic ultrasound found bilateral cystic mass with internal echo sized 50 and 60 mm
in diameter. Her husband sperm examination was within normal limit. From laparoscopy findings,
normal size uterus with adhesion of posterior part to colon and Douglas pouch was also severely
adhered. Both of ovaries were enlarged approximately 50 – 60 mm with severe adhesion at the right
and left adnexae. After adhesiolysis, both of tubes could be identified and were blocked. Chocolate fluid
was spilled from the cysts. The pathology of the cysts was endometriosis cyst. Which one is the TRUE
statement ?
A. The patient should directly undergo surgery to solve her pain and infertility problem
B. Laparoscopic procedures should be done if infertility problem already more than 3 years
C. Laparoscopic cystectomy was done because the cyst have diameter 50 and 60 mm
D. GnRH agonist 2 time prior to surgery will give better results
E. Male factor and ovarian reserve should be measured prior to decision of surgery
38. A parity 3 40-year-old woman complains of cyclical heavy and painful menstrual bleeding. On
examination she is found to have an enlarged globular uterus and a transvaginal sonography revealed
diffuse adenomyosis. She has completed her family and currently relies on condoms for contraception.
She smokes 10 cigarettes per day but is otherwise fit and well. Which treatment would you consider
most appropriate? Choose the single best answer.
A. LNG-IUS
B. COCP
C. Hysterectomy
D. GnRH analogue
E. Endometrial ablation

39. A 26-year-old woman complains of recurrent bouts of bacterial vaginosis (BV) despite
successful initial treatment. She does not douche or smoke and has been in a monogamous relationship
or 6 years. Recurrence of BV after initial treatment is common (up to 30 percent), which can be
frustrating of the patient. Which of the following consistently decreases recurrence rates and should be
recommended to this patient?
A. Treatment of male partners
B. Use of acidifying vaginal gels
C. Probiotics and reintroduction of lactobacilli
D. No intervention consistently decreases recurrence
E. Treatment of long continous antibiotic

40. Patient 65 years old, P4 came to outpatient clinic with chief complaint of frequent urination.
Since 6 months ago she feels the urge to void every hour and also she has to wake up 3-4 times in the
night to void. She never leaks urine. She doesn’t feel any pain during urination and no blood in the
urine. She already came to general practitioner and got antibiotics for 7 days but the symptoms
remained. What is the most useful supporting examination in this patient?

A. Urine culture
B. Pelvic floor ultrasaound
C. Gynecology ultrasound
D. Bladder diary
E. Urodynamic evaluation

41. A 25-year-old woman underwent a uterine curettage after a miscarriage and has had
no menses since. Pregnancy test was negative. Intrauterine adhesions were suspected. Which
of the following is associated with asherman syndrome?
A. Associated with low cortisol levels
B. Associated with low estradiol levels
C. It is associated with class 1 anovulation
D. It is associated with class 2 anovulation
E. Associated with a negative estrogen-progestin test

42. A 34-year old women with primary infertility 3 years, oligomenorrhea and a body mass index
(BMI) of 26. Day 23 progesterone level result was 5 ng/ml. Transvaginal ultrasound shows multiple
small follicle size 5-8 mm in both ovary. HSG shows bilateral patent tubes. Her partner’s semen analysis
show a volume of 3 ml, pH of 7 and a sperm count of 20 million/ml. According to the current
International Guidelines, which of the following is true regarding ovulation induction in PCOS ?
A. Gonadotropin cannot be used as a first line treatment
B. Use of gonadotropin correlate with lower cost of ovulation induction
C. If there are more than two mature follicles ovulation trigger should be cancelled
D. Use of aromatase inhibitor reserved for Clomiphene failure patients
E. Clomiphene citrate has a similar risk of multiple pregnancy compared to aromatase inhibitor

43. Ms N, 37 years old with chief complain of severe dysmenorrhea. Pelvic ultrasound found
diffuse adenomyosis with uterus size 16x10x6cm. There is no history of abnormal uterine bleeding.
Which of the following is the most appropriate management ?
A. Noretisterone 5 mg/day
B. Dienogest 4 mg/day
C. Extended OCP regimen
D. Sequential estrogen-progestin pills
E. LNG IUS

44. A-32-year-old woman presents to your office complaining of prolonged vaginal


bleeding. She has never been pregnant in her 3 years of marriage. Her periods were regular
monthly. Her last normal period was 2 month ago. For the past week she has been bleeding
heavily and passing large clots. On pelvic examination, the external genitalia are
normal. Speculum examination reveals smooth cervix. The uterus is equal with 12 weeks of
pregnancy. The ovaries are normal in palpation. Her urine pregnancy test is negative. Which
of the following is the most likely diagnosis?
A. Cervical polyps
B. Chronic anovulation
C. Coagulation defect
D. Uterine fibroid
E. Endometrioid carcinoma
45. A 40-year-old Female Parity 3 comes to your office to discuss contraception. She has
been married for 15 years and smokes one pack of cigarettes per day. She was diagnosed with
systemic lupus erythematosus (SLE) at the age of 20 years and has only been hospitalized once
for an acute exacerbation of joint swelling and fatigue. Her BMI is 24, and she has light to
normal menstrual flow. She has no plans for a future pregnancy but is not ready to commit to
permanent sterilization. Which of the following condition is NOT an absolute contraindication
to COC pill use?
A. History of or current thromboembolism
B. Coronary artery disease
C. Tobacco use of greater than or equal to 15 cigarettes/day over the age of 35 years
D. Abnormal liver function
E. Diabetes Mellitus
46. A 40-year-old Parity 3 comes to your office to discuss contraception. She has been
married for 15 years and smokes one pack of cigarettes per day. She was diagnosed with
systemic lupus erythematosus (SLE) at the age of 20 years and has only been hospitalized once
for an acute exacerbation of joint swelling and fatigue. Her BMI is 24, and she has light to
normal menstrual flow. She has no plans for a future pregnancy but is not ready to commit to
permanent sterilization. Which of the following are not a reasonable method of contraception
for this patient?
A. Combined oral contraceptive pills
B. Progestin only pills
C. Copper T IUD
D. LNG IUS
E. Progestin injection

47. A 40-year-old Parity 3 comes to your office to discuss contraception. She has been
married for 15 years and smokes one pack of cigarettes per day. She was diagnosed with
systemic lupus erythematosus (SLE) at the age of 20 years and has only been hospitalized once
for an acute exacerbation of joint swelling and fatigue. Her BMI is 24, and she has light to
normal menstrual flow. She has no plans for a future pregnancy but is not ready to commit to
permanent sterilization. After counselling she decided to use progestin only pills. Compared to
users of combination OCs, users of progestin-only OCs (minipills) are less likely to experience
which of the following?
A. Intrauterine pregnancy
B. Irregular vaginal bleeding
C. Gonadotropin suppression
D. Ectopic pregnancies
E. Mood swings

48. A 25 years old woman came with chief complain amenorrhea. She notes that she
diagnosed missed abortion and had a dilatation and curettage (D&C) 4 months ago. After D&C
she had scant vaginal bleeding for a month, and for the last 3 months she notes the absence of
menses but instead has monthly painful cramping. She and her husband would like another
pregnancy. To confirm your suspect diagnosis, which test would you first perform?
A. Provera 10 mg daily to achieve withdrawal bleeding
B. Check Urinary HCG
C. Pelvic Ultrasound examination
D. Dilatation and curretage
E. Check Serum FSH and LH level

49. A 49-year-old woman had a radical hysterectomy and lymph node sampling for stage 1B
squamous cell cancer of the cervix. A suprapubic catheter was placed at the time of surgery. She is now
8 weeks postoperative and has not been able to void. She is also leaking urine with activity, coughing,
and sneezing. What is the most likely reason for voiding difficulty?
A. Spasm of the pelvic floor muscles
B. Outflow obstruction
C. Postoperative swelling around the bladder
D. Innervation to the lower urinary tract was transected
E. Overdistention of the bladder
50. A 26-year-old parity zero woman comes to emergency department complaining of moderate
lower abdominal pain and nausea-vomiting. Her LMP was 1 week ago. She has history of untreated
STIs, with multiple sexual partners and intermittent condom usage as risk factors. On physical
examination, her temperature is 38.4°C. Physical examination shows lower abdominal quadrant
tenderness and mucopurulent cervical discharge and cervical motion tenderness; whereas bilateral
adnexa are unremarkable. On ultrasound examination, you expect to find which of the following:
A. Endometriomas
B. Free fluid in Douglas cavity
C. Thickened, fluid-filled tubes
D. Thickening of the endometrial cavity
E. No remarkable findings

51. A 18 years old girl come to the emergency room with pelvic pain for 2 days. She has low
grade fever. She has regular menstrual cycle. From the physical examination an intense of low
abdominal pain was marked even more on palpation. There was a muscle defence, on the right lower
abdomen quadrant. On abdominal ultrasound reveal a mass measuring 10 cm in the largest diameter on
her pelvic, characterized with multiple hyperechogenic interfaces in a cystic mass in the right ovary.
Uterus anteflexed within normal limit. Left ovary within normal limit. There is no fluid in pelvic cavity.
What would be the working diagnosis?
A. Functional cyst
B. Endometriosis
C. Ovarian torsion
D. An infected cyst
E. Ruptured cyst
52. An 80-year-old woman presents to the office with complaints of leaking large amounts of
urine when she hears running water. Her symptoms began about 1 month ago and have gotten
progressively worse. She denies any previous history of abdominal or pelvic pain. Medical history is
complicated by hypertension treated with hydrochlorothiazide and lisinopril. What is the next best step
in her workup?
A. Urinalysis
B. Urine cytology
C. Urine culture
D. Cystoscopy
E. Basic metabolic panel

53. Ms. S, P0A0, 21 y/o came to the outpatient clinic with chief complain of heavy menstrual
bleeding for almost 2 weeks. Her last menstrual period was 3 months ago. The vital sign was 100/70,
pulse 90x/minutes, respiration 20x/m and the temperature was normal. Her BMI was 35. Her face and
conjuctivas looked pale. The heart and lungs were normal. There is excessive growth of hair in the
lower limbs. The abdominal exam revealed no abdominal mass. There was no pain during abdominal
palpation. During vaginal inspection, some blood cloths were expelled from the vaginal introitus. The
abdominal ultrasound exam found no abnormality. The uterine size was normal. No adnexal mass was
found. The laboratory results showed Hb level of 7,5 gr%. The platelet count was 200.000/mm3 and
the leucocyte count was 10.500/mm3. The pregnancy test result was negative. Which of the following
is the appropriate medication to use for controlling acute and heavy bleeding?
A. GnRH agonist
B. Medoxyprogesterone acetate injection
C. Combined hormonal injection
D. Danazol
E. Conjugated equine estrogen

54. A 33 years old woman with morbid obesity was plan for laparoscopy cystectomy due to
endometriosis cyst. Patient had history of laparotomy cystectomy 2 years before. Regarding her
condition, which of the following that cannot be used to compensate for difficulties during laparoscopy
:
A. Veres needle insertion at an angle that is nearly perpendicular to the skin
B. Steeper Trendelenburg position to improve patient ventilation
C. Placement of additional ancillary ports to assist ini lifting a fattier omentum from the operating
filed
D. Use of a gel pad beneath the patient and tucking of patient to limit patient slippage while in
Trendelenburg position
E. Placement of initial trochar in Lee Huang’s point

55. A 33 years old woman with morbid obesity was plan for laparoscopy cystectomy due to
endometriosis cyst. Patient had history of laparotomy cystectomy 2 years before. Which technique use
to avoid complication for this patient ?
A. Open technique
B. Direct trocar insertion
C. Veress needle
D. Anterior abdominal wall elevation
E. Optical primary trocar

56. A 27 years old patient, P1, come with 7 months of amenorrhea. Her pregnancy test showed
negative result. Her general status was normal. On ultrasound examination revealed hyperechoic, 14
mm endometrial line. On the right ovary there was anechoic mass size 20 mm, left ovary was normal.
Which of the following might be the cause of secondary amenorrhea in this patient ?
A. Anovulatory cycle
B. PCOS
C. Kallman syndrome
D. Primary ovarian insufficiency
E. Central amenorrhea

57. A 27 years old patient, P1, come with 7 months of amenorrhea. Her pregnancy test showed
negative result. Her general status was normal. On ultrasound examination revealed hyperechoic, 14
mm endometrial line. On the right ovary there was anechoic mass size 20 mm, left ovary was normal.
Which of the following should be the next step of management ?
A. Perform endometrial sampling using hysteroscopy
B. Give noretisterone acetate 2x5 mg for 10 days
C. Give combined oral contraception
D. Perform FSH, LH and estradiol examination
E. Perform prolactin examination

58. A 27 years old patient, P1, come with 7 months of amenorrhea. Her pregnancy test showed
negative result. Her general status was normal. On ultrasound examination revealed hyperechoic, 14
mm endometrial line. On the right ovary there was anechoic mass size 20 mm, left ovary was normal.
Which of the following is clinical consequences if this condition left untreated ?
A. Increased risk of ovarian cancer
B. Increased risk of intermenstrual bleeding
C. Increased risk of chronic kidney disease
D. Increased risk of endometrial cancer
E. Increased risk of breast cancer

59. You had a patient, 34 years old with secondary amenorrhoea and based on your examination
you suspected she had premature ovarian inssufiency (POI). What is your background to suspect this
condition?
A. Level of estrogen is 8 pg/ml and FSH is 40 UI/L
B. Level of estrogen is 3 pg/ml and FSH is 20 UI/L
C. She had history of 2 consecutive IVF program
D. She had history of PCOS
E. She had history of Asherman’s syndrome

60. Regarding premature ovarian failure, which statement is true ?


A. Incidence is about 5%
B. It occurs in 50% of patients presenting with secondary amenorrhoea
C. It corresponds to Asherman’s Syndrome
D. Hot flushes occur in about 50% of patients
E. Symptoms are related to the levels of LH and FSH

61. A 18-year-old adolescent female complains of not having started her menses. Her breast
development is Tanner stage II, Pubic hair development was stage I. From vaginal examination
found a small uterus and normal vagina and vulva. Which of the following describes the most
likely diagnosis?
A. Partial androgen insensitivity syndrome
B. Complete androgen insensitivity syndrome
C. Turner syndrome
D. Late onset congenital hyperplasia
E. Polycystic ovarian syndrome
62. A 30 years old patient came with complaint of infertility. Her husband is a 33-year-old who
has had a semen analysis, which was reported as normal. On further history, the patient reports
that her periods have been quiet irregular over the last year and that she has not had period in the
last 3 months. She also reports of weight gain and abnormal hair growth. From ultrasound we
found multiple follicles size 2-8 mm in both ovaries. Which of the following might contribute to
this condition?
A. Low level of LH
B. High level of vitamin D
C. Decreased visceral fat deposition
D. Low level of androgen from the ovary
E. Insulin resistance
63. A 45-year-old presents for evaluation because her primary care physician has diagnosed her
with pelvic organ prolapse while performing annual care. She denies any pelvic pressure, bulge,
or difficulty with urination. Her body mass index is 35 kg/m2. For asymptomatic grade 1 pelvic
organ prolapse, what do you recommend?
A. Pelvic floor muscle exercises
B. Weight loss program
C. Laser vaginal rejuvenation
D. Pessary
E. Reconstructive surgery

64. A 38-year-old woman presents to the office with a six months history of low pelvic pain. Which
of the following is the LEAST likely cause?
A. Interstitial cystitis
B. Adenomysosis
C. Ectopic pregnancy
D. Pelvic inflammatory disease
E. Inflammatory bowel disease

65. A 38-year-old woman presents to the office with a six months history of low pelvic pain. If on
physical examination reveal a right adnexal pain and also accompany with right upper quadrant
abdominal pain, what is the most likely etiology of this condition?
A. Cholecystitis
B. Hepatitis
C. Hepatic abscess
D. Appendicitis
E. Fitz-Hugh Curtis syndrome

66. A 52 years old woman came to the ER with complaint of shortness of breath since 2 days
ago. She also noticed an abdominal enlargement since 6 months ago. If on Ultrasound
examination reveal a. regular 7 cm solid tumor with non-vascular appearance and ascites. And
on chest X-Ray reveal a right moderate pleural effusion with no abnormality in lung parenchym.
What is the most likely diagnosis of this condition?
A. Mature cystic teratoma
B. Malignant ovarian tumor
C. Meigs syndrome
D. Colon cancer
E. Tuberculosis
67. A 52 years old woman came to the ER with complaint of shortness of breath since 2 days ago.
She also noticed an abdominal enlargement since 6 months ago. If on Ultrasound examination
reveal a. regular 7 cm solid tumor with non-vascular appearance and ascites. And on chest X-
Ray reveal a right moderate pleural effusion with no abnormality in lung parenchym. Which of
the following non-malignant adnexal masses can give rise to this condition?
A. Endometrioma
B. Mature cystic teratoma
C. Intra-ligament Leiomyoma
D. Ovarian fibroma
E. Mucinous cystadenoma

68. Gin A 30 years old patient came with complaint of infertility. Her husband is a 33-year-old who
has had a semen analysis, which was reported as normal. On further history, the patient reports
that her periods have been quiet irregular over the last year and that she has not had period in
the last 3 months. She also reports of weight gain and abnormal hair growth. From ultrasound
we found multiple follicles size 2-8 mm in both ovaries. Which of the following correlate to
this condition?

A) Day 3 FSH level 20 IU

B) Serum AMH level 2,6 ng/ml

C) Positive Clomiphene citrate challenge test

D) Midluteal progesterone level of 4 ng/ml

E) Follicle antral basal count of 12

69. Gin A 28-year-old woman is hoping to become pregnant soon. She is worried about her history of
acute pelvic inflammatory disease (PID) when in college 8 years ago. Which of the following is
the pathogenesis of infertility due to history of PID?

A) Direct destruction of tubal mucosa by chlamydia

B) Inflammatory response followed with fibrosis to protein released by chlamydia

C) Inflammation of the endometrium

D) Ovulatory dysfunction due to pelvic inflammatory disease

E) Inflammation that hinders spermatozoa movement inside the uterus

70. Gin A 72 years old woman P6 came to outpatient clinic with chief complaint of bulging mass
protrudes from vagina since 3 months ago. The mass usually occurs during activity and also
when she defecate , and disappear when lying down. There were no difficulty in voiding and
defecation. No urinary leakage during coughing and sneezing. She is not sexually active. If on
the Pelvic Organ Prolapse Quantification examination result showing below. What is the
diagnosis of this patient?
A) Uterine prolapse grade 2, cystocele grade 2, rectocele grade 1

B) Uterine prolapse grade 4, cystocele grade 3, rectocele grade 2

C) Uterine prolapse grade 2, cystocele grade 3, rectocele grade 2

D) Uterine prolapse grade 3, cystocele grade 3, rectocele grade 2

E) Uterine prolapse grade 3, cystocele grade 2, rectocele grade 2

71. Gin A 45 years old woman presents to your office for consultation regarding her symptoms of
menopause. She stopped having periods 13 months ago after BSO operation and is having
severe hot flushes. She is considering hormonal therapy to alleviate it. Which of the following
is true regarding menopause condition?

A) Hormonal therapy are contraindicated in patient with chronic kidney disease

B) Combine E-P hormonal therapy has a lower risk of breast cancer in compared to Estrogen only
therapy

C) Combined oral contraception is a first line therapy for menopause

D) Early intervention by giving hormone therapy will lower risk of CVD in menopause patients

E) Hormonal therapy in menopause patient has a higher risk of colorectal cancer

72. Gin A patient present to you with pain and swelling in the vulva. On examination you find a
reddish bulge on the vaginal introitus at 4 o'clock positions sized 3 cm, cystic and pain on
palpation.What treatment of choice for this condition?

A) Antibiotic for 7 days

B) Incision and drainage

C) Marsupialization

D) Cystectomy
E) Excision of the glands

73. Mrs. N, 39 years old had just undergone laparoscopic procedures. Her chief complaint was
infertility for 6 years with history of severe dysmenorrhea. Intraoperative findings were patent
both tubes and AFS stage III-IV endometriosis. Which of the following is true regarding
management in this patient?

A) GnRH agonist 3 time prior to surgery will give better results

B) Medical management with GnRH agonist should be given for 3 months followed with In Vitro
Fertilization

C) Patient should be given secondary prevention with oral progestin Dienogest 2 mg/day

D) Intrauterine insemination is the preferred methods of infertility treatment in this case

E) AMH level must be measured before starting fertility treatment after surgery

74. A parity 3 40-year-old woman complains of cyclical heavy and painful menstrual bleeding. On
examination she is found to have an enlarged globular uterus and a transvaginal sonography
revealed diffuse adenomyosis. She has completed her family and currently relies on condoms
for contraception. She smokes 10 cigarettes per day but is otherwise fit and well. Choose the
single best answer, which treatment would you consider most appropriate?

A) LNG-IUS

B) COCP

C) Hysterectomy

D) GnRH analogue

E) Endometrial ablation

75. Patient 65 years old, P4 came to outpatient clinic with chief complaint of frequent urination.
Since 6 months ago she feels the urge to void every hour and also she has to wake up 3-4 times
in the night to void. She never leaks urine. She doesn't feel any pain during urination and no
blood in the urine. She already came to general practitioner and got antibiotics for 7 days but
the symptoms remained. What is the most useful supporting examination in this patient?

A) Urine culture

B) Pelvic floor ultrasound

C) Gynecology ultrasound

D) Bladder diary

E) Urodynamic evaluation
76. 60Gin A 25-year-old woman underwent a uterine curettage after a miscarriage and has had no
menses since. Pregnancy test was negative. Intrauterine adhesions were suspected. Which of
the following is associated with asherman syndrome?

A) Associated with low cortisol levels

B) Associated with low estradiol levels

C) It is associated with anovulation

D) It is commonly caused by curettage due to miscarriage

E) It is classified into 6 types according to American Fertility Society

77. 62Gin Mrs. N, 27 years old with chief complain of infertility for 2 years with history of severe
dysmenorrhea. From hysterosalpingography, both tubes were patent. Pelvic ultrasound found
bilateral cystic mass with internal echo sized 25 and 40 mm in diameter, her antral follicle
count was 12. Her husband sperm examination was within normal limit. What is the next
appropriate management?

A) Offer her IUI

B) Perform laparoscopy cystectomy and adhesiolysis

C) Give GnRH analog for 3 months continue with IUI

D) Give Dienogest 1x2mg for 6 months

E) Offer her IVF

78. 64Gin A-32-year-old woman presents to your office complaining of prolonged vaginal
bleeding. She has never been pregnant in her 3 years of marriage. Her periods were regular
monthly. Her last normal period was 2 month ago. For the past week she has been bleeding
heavily and passing large clots. On pelvic examination, the external genitalia are
normal. Speculum examination reveals smooth cervix. The uterus is equal with 12 weeks of
pregnancy. The ovaries are normal in palpation. Her urine pregnancy test is negative. Which
type of pathology that can cause this kind of complaint?

A) Cervical polyps

B) Chronic anovulation

C) Coagulation defect

D) Uterine fibroid FIGO type 6-7

E) Uterine fibroid FIGO type 2-5

79. A 25 years old woman came with chief complain amenorrhea. She notes that she diagnosed
missed abortion and had a dilatation and curettage (D&C) 4 months ago. After D&C she had
scant vaginal bleeding for a month, and for the last 3 months, she notes the absence of menses
but instead has monthly painful cramping. She and her husband would like another pregnancy.
Her urinary HCG test was negative. To confirm your suspect diagnosis, which test would you
first perform?

A) Provera 10 mg daily to achieve withdrawal bleeding

B) Check blood HCG examination

C) Pelvic Ultrasound examination

D) Dilatation and curretage

E) Check Serum FSH and LH level

80. A 26-year-old parity zero woman comes to emergency department complaining of moderate
lower abdominal pain and nausea-vomiting. She has trouble in her oral intake and has
difficulties of taking medication. Her LMP was 1 week ago. She has history of untreated STIs,
with multiple sexual partners and intermittent condom usage as risk factors. On physical
examination, her temperature is 38.4°C. Physical examination shows lower abdominal quadrant
tenderness and mucopurulent cervical discharge and cervical motion tenderness; whereas
bilateral adnexa are unremarkable. Which of the following antibiotics is the drug of choice for
this patient?

A) Ampicillin sulbactam 2x375 mg orally

B) Metronidazol 3x500 mg orally

C) Ceftriaxone injection 2x1 gram/day

D) Clindamycin injection 2x300mg/day

E) Meropenem 3x1 gram /day

81. 71Gin A 18 years old girl come to the emergency room with pelvic pain for 2 days. She has low
grade fever. She has regular menstrual cycle. From the physical examination an intense of low
abdominal pain was marked even more on palpation. There was a muscle defence, on the right
lower abdomen quadrant. On abdominal ultrasound reveal a mass measuring 10 cm in the
largest diameter on her pelvic, characterized with multiple hyperechogenic interfaces in a cystic
mass in the right ovary. Uterus anteflexed within normal limit. Left ovary within normal limit.
There is no fluid in pelvic cavity. What would be the working diagnosis?

A) Functional cyst

B) Endometriosis

C) Ovarian torsion

D) An infected cyst

E) Ruptured cyst
82. 73Gin A 56-year-old lady presents to gynecology outpatient clinic with postmenopausal
bleeding. A thorough clinical workup reveals an endometrial thickness of 7 mm with a regular
basement membrane. She underwent hysteroscopy biopsy, and the histopathology result was
simple hyperplasia without atypia. What is the best management option for this patient?

A) Curretage

B) LNG-IUS

C) Endometrial ablation

D) Total hysterectomy

E) Total hysterectomy and bilateral salpingo-oophorectomy

83. Ms. S, P0A0, 21 y/o came to the outpatient clinic with chief complain of heavy menstrual
bleeding for almost 2 weeks. Her last menstrual period was 3 months ago. The vital sign was
100/70, pulse 90x/minutes, respiration 20x/m and the temperature was normal. Her BMI was
35. Her face and conjuctivas looked pale. The heart and lungs were normal. There is excessive
growth of hair in the lower limbs. The abdominal exam revealed no abdominal mass. There was
no pain during abdominal palpation. During vaginal inspection, some blood cloths were
expelled from the vaginal introitus. From ultrasound examination revealed normal uterus and
both adnexas. The laboratory results showed Hb level of 9 gr/dl. The platelet count was
200.000/mm3 and the leucocyte count was 10.500/mm3. The pregnancy test result was
negative .Which of the following is the appropriate medication to use for controlling acute and
heavy bleeding?

A) GnRH agonist

B) Medoxyprogesterone acetate injection

C) Combined hormonal injection

D) Danazol

E) Noretisterone

84. A 33 years old woman with morbid obesity was plan for laparoscopy cystectomy due to
endometriosis cyst. Patient had history of laparotomy cystectomy 2 years before. Regarding her
condition, which of the following that cannot be used to compensate for difficulties during
laparoscopy ?

A) Veres needle insertion at an angle that is nearly perpendicular to the skin

B) Steeper Trendelenburg position to improve patient ventilation

C) Placement of additional ancillary ports to assist ini lifting a fattier omentum from the operating
filed

D) Use of a gel pad beneath the patient and tucking of patient to limit patient slippage while in
Trendelenburg position
E) Placement of initial trochar in Lee Huang’s point

85. A 33 years old woman with morbid obesity was plan for laparoscopy cystectomy due to
endometriosis cyst. Patient had history of laparotomy cystectomy 2 years before. Which
technique use to avoid complication for this patient?

A) Open technique

B) Direct trocar insertion

C) Veress needle

D) Anterior abdominal wall elevation

E) Optical primary trocar

86. A 27 years old patient, P1, come with 7 months of amenorrhea. Her pregnancy test showed
negative result. Her general status was normal. On ultrasound examination revealed
hyperechoic, 14 mm endometrial line. On the right ovary there was anechoic mass size 20 mm,
left ovary was normal. Which of the following might be the cause of secondary amenorrhea in
this patient?

A) Anovulatory cycle

B) PCOS

C) Kallman syndrome

D) Primary ovarian insufficiency

E) Central amenorrhea

87. A 27 years old patient, P1, come with 7 months of amenorrhea. Her pregnancy test showed
negative result. Her general status was normal. On ultrasound examination revealed
hyperechoic, 14 mm endometrial line. On the right ovary there was anechoic mass size 20 mm,
left ovary was normal.Which of the following should be the next step of management?

A) Perform endometrial sampling using hysteroscopy

B) Give noretisterone acetate 2x5 mg for 10 days

C) Give combined oral contraception

D) Perform FSH, LH and estradiol examination

E) Perform prolactin examination

88. A 27 years old patient, P1, come with 7 months of amenorrhea. Her pregnancy test showed
negative result. Her general status was normal. On ultrasound examination revealed
hyperechoic, 14 mm endometrial line. On the right ovary there was anechoic mass size 20 mm,
left ovary was normal. Which of the following is clinical consequences if this condition left
untreated?

A) Increased risk of ovarian cancer

B) Increased risk of intermenstrual bleeding

C) Increased risk of chronic kidney disease

D) Increased risk of endometrial cancer

E) Increased risk of breast cancer

89. You had a patient, 34 years old with secondary amenorrhoea and based on your examination
you suspected she had premature ovarian insufficiency (POI). Her FSH level is 40 IU/L and
estradiol level is 5 pg/ml. What is the best medication to be used in this patient?

A) Combined oral contraceptive pill ethinil estradiol 30 ug/drospirenone 2 mg

B) Conjugated equine estrogen 0.625 mg/day continuously

C) Estradiol valerate 2 mg/day continuously

D) Sequential estradiol valerate 1 mg with drospirenone 2 mg

E) Combine injectable contraception monthly

90. Regarding premature ovarian failure, which statement is true ?

A) Incidence is about 5%

B) It occurs in 50% of patients presenting with secondary amenorrhoea

C) It corresponds to Asherman's Syndrome

D) Hot flushes occur in about 50% of patients

E) Symptoms are related to the levels of LH and FSH

91. A 18-year-old adolescent female complains of not having started her menses. Her breast
development is Tanner stage I, Pubic hair development was stage I. From vaginal examination
found a small uterus and normal vagina and vulva. Which of the following describes the most
likely diagnosis?

A) Partial androgen insensitivity syndrome

B) Complete androgen insensitivity syndrome

C) Gonadal dysgenesis

D) Late onset congenital hyperplasia

E) Swyer syndrome
92. A 30 years old patient came with complaint of infertility. Her husband is a 33-year-old who has
had a semen analysis, which was reported as normal. On further history, the patient reports that
her periods have been quiet irregular over the last year and that she has not had period in the
last 3 months. She also reports of weight gain and abnormal hair growth. From ultrasound we
found multiple follicles size 2-8 mm in both ovaries. Which of the following might contribute
to this condition?

A) Low level of LH

B) High level of vitamin D

C) Decreased visceral fat deposition

D) Low level of androgen from the ovary

E) Insulin resistance

93. A 28 years old patient, P0, presents to your clinic for fertility workup. She had been married for
2 years with regular intercourse. Her menstrual cycle is normal. Her general status was normal.
Vaginal examination revealed normal findings. The following month she came back with the
result of hysterosalpingography (see the picture below).What will be your next step ?

A) Order Semen Analysis

B) Schedule operative laparoscopy

C) Gives clomiphene citrate and plan for natural conception

D) Gives clomiphene citrate and plan for intrauterine insemination

E) Plan for IVF


94. 86Gin A 38-year-old woman presents to the office with a six months history of low pelvic pain.
Which of the following is the LEAST likely cause?

A) Interstitial cystitis

B) Adenomysosis

C) Ectopic pregnancy

D) Pelvic inflammatory disease

E) Inflammatory bowel disease

95. 52 years old woman came to the ER with a complaint of shortness of breath since 2 days ago.
She also noticed an abdominal enlargement since 6 months ago. On physical examination found
tachypnea and chest examination reveal dullness to percussion on lower right hemithorax and
on abdominal examination found shifting dullness and a solid mass on pelvic examination.
What is the most likely diagnosis in this case?

A) Mature cystic teratoma

B) Malignant ovarian tumor

C) Meigs syndrome

D) Colon cancer

E) Tuberculosis

96. A 52 years old woman came to the ER with complaint of shortness of breath since 2 days ago.
She also noticed an abdominal enlargement since 6 months ago. If on Ultrasound examination
reveal a. regular 7 cm solid tumor with non-vascular appearance and ascites. And on chest X-
Ray reveal a right moderate pleural effusion with no abnormality in lung parenchym. What is
the most likely diagnosis of this condition?

A) Mature cystic teratoma

B) Malignant ovarian tumor

C) Meigs syndrome

D) Colon cancer

E) Tuberculosis

97. A 52 years old woman came to the ER with complaint of shortness of breath since 2 days ago.
She also noticed an abdominal enlargement since 6 months ago. If on Ultrasound examination
reveal a. regular 7 cm solid tumor with non-vascular appearance and ascites. And on chest X-
Ray reveal a right moderate pleural effusion with no abnormality in lung parenchym. Which of
the following non-malignant adnexal masses can give rise to this condition?
A) Endometrioma

B) Mature cystic teratoma

C) Intra-ligament Leiomyoma

D) Ovarian fibroma

E) Mucinous cystadenoma

98. A hirsute obese, 24-year-old woman presents with irregular periods. Pelvic sonogram
demonstrates bilateral ovaries with multiple peripheral follicles. Which is the most likely
laboratory result from this patient?

A) High estradiol level

B) High progesterone level

C) High FSH level

D) High LH level

E) High prolactin level

99. A hirsute obese, 24-year-old woman presents with irregular periods. Pelvic sonogram
demonstrates bilateral ovaries with multiple peripheral follicles. From laboratory examination
revealed elevated testosterone levels. Which of the following structure can be the production
source of androgen?

A) Anterior pituitary

B) External theca cells

C) Internal theca cells

D) Granulosa cells

E) Adrenal gland medulla

100. A hirsute obese, 24-year-old woman presents with irregular periods. Pelvic sonogram
demonstrates bilateral ovaries with multiple peripheral follicles. What is the most appropriate
management for this patient?

A) Observation

B) GnRH agonist

C) Combine oral contraceptive pills

D) Surgical management

E) Metformin
101. A patient presents with vulvar irritation and pruritis. Speculum examination reveals a
hyperemic, edematous, vaginal vault with odorless discharge. The pH is 4.0. These symptoms
had been recurrent more than 4 times during the last 1 year. What is the most likely the cause of
this condition?

A) Bacterial vaginosis

B) Trichomonas. Vaginalis

C) Neisseria Gonorrhoea

D) Candida sp.

E) Chlamydia trachomatis

102. A 32-year-old woman presents to your office. She complained about her sexual problems
of pain during sexual intercourse since giving spontaneous birth of her second child 6 months
ago. She feels a normal desire to engaged in sexual intercourse. She is now still breastfeed the
baby and she is in combine oral contraceptive pills. What is the most common cause of this
problem?

A) Infection

B) Episotomy scar

C) Combine Oral contraceptive pills

D) Fatigue

E) Breastfeeding

103. A patient presents with vulvar irritation and pruritis. Speculum examination reveals a
hyperemic, edematous, vaginal vault with odorless discharge. The pH is 4.0. These symptoms
had been recurrent more than 4 times during the last 1 year. What is the treatment of choice for
this condition?

A) Single dose of 150 mg of fluconazole

B) Single dose of 2000 mg metronidazole

C) Fluconazole 150 mg every alternate day for 3 doses

D) 2000 mg of metronidazole weekly for 6 months

E) Fluconazole 150 mg weekly for 6 months

103. A 60-year-old P2 presents to the urogynecology clinic with complaints of urinary incontinence.
She has urinary urgency and can’t make it to the bathroom before leaking a large amount of
urine. She urinate 12 time during the day and gets up two to three times per night to
urinate. She also complaint of urinary leakage during coughing and sneezing. If the urinalysis
revealed normal, what is the next diagnostic plan?
A) Post void residual urine measurement
B) Pad test
C) Voiding Diary
D) Pelvic floor ultrasound
E) E) Urodynamic study

104. Your patient is a 13-year-old adolescent girl who presents with cyclic pelvic pain. She has
never had a menstrual cycle. She is afebrile and her vital signs are stable. On physical
examination, she has age-appropriate breast and pubic hair development and normal external
genitalia. there is no bulge in the introitus of vagina and no hymenal structure seen. You obtain
a transabdominal ultrasound, which reveals a hematocolpos and hematometra. What is the most
likely diagnosis?

A) Distal Transverse vaginal septum

B) Longitudinal vaginal septum

C) Imperforate hymen

D) Distal Vaginal agenesis

E) MRKH

105. A 89-year-old female patient with multiple, serious medical comorbidities presents to discuss
options for treatment of her high-grade prolapse. The prolapse is externalized and becoming
ulcerated from friction against her undergarments. Her main priority is to “fix or get rid of this
thing,” but her primary care provider has cautioned against a lengthy or open abdominal
procedure. She is not interested in future intercourse. What can you offer this patient?

A) Total colpocleisis

B) Partial colpocleisis

C) Sacrospinous hysteropexy

D) Ring pessary.

E) Manchester Fothergyl operation

106. A 38 years old multi gravid woman complains of the painless loss of urine, beginning
immediately with coughing, laughing, lifting, or straining. cessation of the activity do not stops
the urine loss. She also complains of frequency with small-voided volume of urine, but no
urgency. This history is most suggestive of ?

A) Fistula

B) Stress incontinence

C) Urge incontinence

D) Overflow incontinence
E) Mixed incontinence

107. A 30 years old patient came with complaint of infertility. Her husband is a 33-year-old who
has had a semen analysis, which was reported as normal. On further history, the patient reports
that her periods have been quiet irregular over the last year and that she has not had period in
the last 3 months. She also reports of weight gain and abnormal hair growth. From ultrasound
we found multiple follicles size 2-8 mm in both ovaries. Which of the following correlate to this
condition?

A) Day 3 FSH level 20 IU

B) Serum AMH level 2,6 ng/ml

C) Positive Clomiphene citrate challenge test

D) Midluteal progesterone level of 4 ng/ml

E) Follicle antral basal count of 12

108. A 45 years old woman presents to your office for consultation regarding her symptoms of
menopause. She stopped having periods 13 months ago after BSO operation and is having
severe hot flushes. She is considering hormonal therapy to alleviate it. Which of the following is
true regarding menopause condition?

A) Hormonal therapy are contraindicated in patient with chronic kidney disease

B) Combine E-P hormonal therapy has a lower risk of breast cancer in compared to Estrogen only
therapy

C) Combined oral contraception is a first line therapy for menopause

D) Early intervention by giving hormone therapy will lower risk of CVD in menopause patients

E) Hormonal therapy in menopause patient has a higher risk of colorectal cancer

109. A patient present to you with pain and swelling in the vulva. On examination you find a
reddish bulge on the vaginal introitus at 4 o'clock positions sized 3 cm, cystic and pain on
palpation.What treatment of choice for this condition?

A) Antibiotic for 7 days

B) Incision and drainage

C) Marsupialization

D) Cystectomy

E) Excision of the glands


110. Mrs. N, 39 years old had just undergone laparoscopic procedures. Her chief complaint was
infertility for 6 years with history of severe dysmenorrhea. Intraoperative findings were patent
both tubes and AFS stage III-IV endometriosis. Which of the following is true regarding
management in this patient?

A) GnRH agonist 3 time prior to surgery will give better results

B) Medical management with GnRH agonist should be given for 3 months followed with In Vitro
Fertilization

C) Patient should be given secondary prevention with oral progestin Dienogest 2 mg/day

D) Intrauterine insemination is the preferred methods of infertility treatment in this case

E) AMH level must be measured before starting fertility treatment after surgery

111. A parity 3 40-year-old woman complains of cyclical heavy and painful menstrual bleeding.
On examination she is found to have an enlarged globular uterus and a transvaginal sonography
revealed diffuse adenomyosis. She has completed her family and currently relies on condoms for
contraception. She smokes 10 cigarettes per day but is otherwise fit and well. Choose the single
best answer, which treatment would you consider most appropriate?

A) LNG-IUS

B) COCP

C) Hysterectomy

D) GnRH analogue

E) Endometrial ablation

112. A 31-year-old women come to outpatient clinic to discuss about her laboratory result.
You tell the patient that she has tested positive for a Chlamydia infection. She tells you
that her friend had chlamydia and told her that it causes a weird vaginal discharge. She
questions the result saying she didn’t have any weird discharge. What would be the best
response?
A) The false positivity rate of the examination is around 10%
B) The majority of patients with chlamydia are asymptomatic
C) Vaginal discharge is not actually a symptom of chlamydia
D) Patient can take a NAATs test from urine sample to confirm the diagnosis
E) Vaginal discharge only occurs when chlamydia progresses to pelvic inflammatory disease

113. Ms N, 37 years old with chief complain of severe dysmenorrhea. Pelvic ultrasound
found diffuse adenomyosis with uterus size 16x10x6cm. There is no history of abnormal uterine
bleeding. Which of the following is the most appropriate management?

A) Noretisterone 5 mg/day
B) Dienogest 4 mg/day

C) Extended OCP regimen

D) Sequential estrogen-progestin pills

E) LNG IUS

114. A 40-year-old Female Parity 3 comes to your office to discuss contraception. She has been
married for 15 years and smokes one pack of cigarettes per day. She was diagnosed with
systemic lupus erythematosus (SLE) at the age of 20 years and has only been hospitalized once
for an acute exacerbation of joint swelling and fatigue. Her BMI is 24, and she has light to normal
menstrual flow. She has no plans for a future pregnancy but is not ready to commit to
permanent sterilization. Which of the following condition is NOT an absolute contraindication to
COC pill use?

A) History of or current thromboembolism

B) Coronary artery disease

C) Tobacco use of greater than or equal to 15 cigarettes/day over the age of 35 years

D) History of breast lump

E) Abnormal liver function

115. A 40-year-old Parity 3 comes to your office to discuss contraception. She has been married
for 15 years and never smokes cigarette. She had a history of repeated preeclampsia in all of her
pregnancies before. She was diagnosed with systemic lupus erythematosus (SLE) at the age of 20
years with history of musculoskeletal and hematologic flare. Her BMI is 26, haemoglobin level is
9,9 g/dl. Which of the following method is the preferred method of contraception?

A) COC pills

B) Progestin only pills

C) Copper T IUD

D) Progestin injection

E) Tubal sterilization

116. A 40-year-old Parity 3 comes to your office to discuss contraception. She has been married
for 15 years and smokes one pack of cigarettes per day. Her BMI is 24, and she has light to
normal menstrual flow. She has no plans for a future pregnancy but is not ready to commit to
permanent sterilization. After counselling, she decided to use progestin-only pills. Which of the
following is true regarding drospirenone only pill?

A) It is given in a 21-7 regiment

B) Daily dose is drospirenone 2 mg/day


C) It has a better bleeding profile compared to other progestin only pills

D) It can suppress follicle growth and prevent ovulation

E) It has a lower risk for ectopic pregnancy

117. A 25 years old woman came with chief complain amenorrhea. She notes that she diagnosed
missed abortion and had a dilatation and curettage (D&C) 4 months ago. After D&C she had
scant vaginal bleeding for a month, and for the last 3 months, she notes the absence of menses
but instead has monthly painful cramping. She and her husband would like another pregnancy.
Her urinary HCG test was negative. To confirm your suspect diagnosis, which test would you first
perform?

A) Provera 10 mg daily to achieve withdrawal bleeding

B) Check blood HCG examination

C) Pelvic Ultrasound examination

D) Dilatation and curretage

E) Check Serum FSH and LH level

118. A 49-year-old woman had a radical hysterectomy and lymph node sampling for stage 1B
squamous cell cancer of the cervix. A suprapubic catheter was placed at the time of surgery. She
is now 8 weeks postoperative and has not been able to void. She is also leaking urine with
activity, coughing, and sneezing. What is the most likely diagnosis of this patient?

A) Vesicovaginal fistula

B) Urge incontinence

C) Stress incontinence

D) Overflow incontinence

E) Mixed incontinence

119. A 26-year-old parity zero woman comes to emergency department complaining of


moderate lower abdominal pain and nausea-vomiting. She has trouble in her oral intake and has
difficulties of taking medication. Her LMP was 1 week ago. She has history of untreated STIs,
with multiple sexual partners and intermittent condom usage as risk factors. On physical
examination, her temperature is 38.4°C. Physical examination shows lower abdominal quadrant
tenderness and mucopurulent cervical discharge and cervical motion tenderness; whereas
bilateral adnexa are unremarkable. Which of the following antibiotics is the drug of choice for
this patient?

A) Ampicillin sulbactam 2x375 mg orally

B) Metronidazol 3x500 mg orally


C) Ceftriaxone injection 2x1 gram/day

D) Clindamycin injection 2x300mg/day

E) Meropenem 3x1 gram /day

120. A 18 years old girl come to the emergency room with pelvic pain for 2 days. She has low
grade fever. She has regular menstrual cycle. From the physical examination an intense of low
abdominal pain was marked even more on palpation. There was a muscle defence, on the right
lower abdomen quadrant. On abdominal ultrasound reveal a mass measuring 10 cm in the
largest diameter on her pelvic, characterized with multiple hyperechogenic interfaces in a cystic
mass in the right ovary. Uterus anteflexed within normal limit. Left ovary within normal limit.
There is no fluid in pelvic cavity. What would be the working diagnosis?

A) Functional cyst

B) Endometriosis

C) Ovarian torsion

D) An infected cyst

E) Ruptured cyst

121. An 80-year-old woman presents to the office with complaints of leaking large amounts of
urine when she hears running water. Her symptoms began about 1 month ago and have gotten
progressively worse. She denies any previous history of abdominal or pelvic pain. Medical history
is complicated by hypertension treated with hydrochlorothiazide and lisinopril. If the urinalysisi
revealed normal, what is the next best step in her workup?

A) Voiding diary

B) Urine cytology

C) Urine culture

D) Cystoscopy

E) Basic metabolic panel

122. A 56-year-old lady presents to gynecology outpatient clinic with postmenopausal bleeding. A
thorough clinical workup reveals an endometrial thickness of 7 mm with a regular basement
membrane. She underwent hysteroscopy biopsy, and the histopathology result was simple
hyperplasia without atypia. What is the best management option for this patient?

A) Curretage

B) LNG-IUS

C) Endometrial ablation

D) Total hysterectomy
E) Total hysterectomy and bilateral salpingo-oophorectomy

123. Ms. S, P0A0, 21 y/o came to the outpatient clinic with chief complain of heavy menstrual
bleeding for almost 2 weeks. Her last menstrual period was 3 months ago. The vital sign was
100/70, pulse 90x/minutes, respiration 20x/m and the temperature was normal. Her BMI was
35. Her face and conjuctivas looked pale. The heart and lungs were normal. There is excessive
growth of hair in the lower limbs. The abdominal exam revealed no abdominal mass. There was
no pain during abdominal palpation. During vaginal inspection, some blood cloths were expelled
from the vaginal introitus. From ultrasound examination revealed normal uterus and both
adnexas. The laboratory results showed Hb level of 9 gr/dl. The platelet count was
200.000/mm3 and the leucocyte count was 10.500/mm3. The pregnancy test result was
negative .Which of the following is the appropriate medication to use for controlling acute and
heavy bleeding?

A) GnRH agonist

B) Medoxyprogesterone acetate injection

C) Combined hormonal injection

D) Danazol

E) Noretisterone

124. A 33 years old woman with morbid obesity was plan for laparoscopy cystectomy due to
endometriosis cyst. Patient had history of laparotomy cystectomy 2 years before. Which
technique use to avoid complication for this patient?

A) Open technique

B) Direct trocar insertion

C) Veress needle

D) Anterior abdominal wall elevation

E) Optical primary trocar

125. A 27 years old patient, P1, come with 7 months of amenorrhea. Her pregnancy test showed
negative result. Her general status was normal. On ultrasound examination revealed
hyperechoic, 14 mm endometrial line. On the right ovary there was anechoic mass size 20 mm,
left ovary was normal.Which of the following should be the next step of management?

A) Perform endometrial sampling using hysteroscopy

B) Give noretisterone acetate 2x5 mg for 10 days

C) Give combined oral contraception

D) Perform FSH, LH and estradiol examination


E) Perform prolactin examination

126. A 27 years old patient, P1, come with 7 months of amenorrhea. Her pregnancy test showed
negative result. Her general status was normal. On ultrasound examination revealed
hyperechoic, 14 mm endometrial line. On the right ovary there was anechoic mass size 20 mm,
left ovary was normal. Which of the following is clinical consequences if this condition left
untreated?

A) Increased risk of ovarian cancer

B) Increased risk of intermenstrual bleeding

C) Increased risk of chronic kidney disease

D) Increased risk of endometrial cancer

E) Increased risk of breast cancer

127. A 18-year-old adolescent female complains of not having started her menses. Her breast
development is Tanner stage I, Pubic hair development was stage I. From vaginal examination
found a small uterus and normal vagina and vulva. Which of the following describes the most
likely diagnosis?

A) Partial androgen insensitivity syndrome

B) Complete androgen insensitivity syndrome

C) Gonadal dysgenesis

D) Late onset congenital hyperplasia

E) Swyer syndrome

128. A 28 years old patient, P0, presents to your clinic for fertility workup. She had been married
for 2 years with regular intercourse. Her menstrual cycle is normal. Her general status was
normal. Vaginal examination revealed normal findings. The following month she came back with
the result of hysterosalpingography (see the picture below).What will be your next step ?
A) Order Semen Analysis

B) Schedule operative laparoscopy

C) Gives clomiphene citrate and plan for natural conception

D) Gives clomiphene citrate and plan for intrauterine insemination

E) Plan for IVF

128. A 57-year-old presents with complain of protruding mass came out from vagina for 6
months. She has difficulty in urination. On physical examination found a grade 3 cystocele grade
2 uterine prolapse and grade 2 rectocele. Which structure that correspond to pathophysiology of
Cystocele?
A) Cardinal ligament
B) Uterosacral ligament
C) Levator Ani muscle
D) Endoplevic fascia
E) pubocervical fascia

129. A 38-year-old woman presents to the office with sudden onset of pelvic pain. If on physical
examination reveal a right adnexal mass with tenderness. She has no history of fever or cervical
discharge. From ultrasound examination revealed right adnexal mass with whirlpool appearance
from doppler examination. What is the most likely cause of this condition?

A) Ruptured of endometriosis cyst

B) Infected ovarian cyst

C) Appendicitis

D) Ovarian torsion

E) Fitz-Hugh Curtis syndrome


130. 52 years old woman came to the ER with a complaint of shortness of breath since 2 days
ago. She also noticed an abdominal enlargement since 6 months ago. On physical examination
found tachypnea and chest examination reveal dullness to percussion on lower right hemithorax
and on abdominal examination found shifting dullness and a solid mass on pelvic examination.
What is the most likely diagnosis in this case?

A) Mature cystic teratoma

B) Malignant ovarian tumor

C) Meigs syndrome

D) Colon cancer

E) Tuberculosis

131. A hirsute obese, 24-year-old woman presents with irregular periods. Pelvic sonogram
demonstrates bilateral ovaries with multiple peripheral follicles. From laboratory examination
revealed elevated testosterone levels. Which of the following structure can be the production
source of androgen?

A) Anterior pituitary

B) External theca cells

C) Internal theca cells

D) Granulosa cells

E) Adrenal gland medulla

132. A hirsute obese, 24-year-old woman presents with irregular periods. Pelvic sonogram
demonstrates bilateral ovaries with multiple peripheral follicles. What is the most appropriate
management for this patient?

A) Observation

B) GnRH agonist

C) Combine oral contraceptive pills

D) Surgical management

E) Metformin

133. A patient presents with vulvar irritation and pruritis. Speculum examination reveals a
hyperemic, edematous, vaginal vault with odorless discharge. The pH is 4.0. These symptoms
had been recurrent more than 4 times during the last 1 year. What is the treatment of choice for
this condition?
A) Single dose of 150 mg of fluconazole

B) Single dose of 2000 mg metronidazole

C) Fluconazole 150 mg every alternate day for 3 doses

D) 2000 mg of metronidazole weekly for 6 months

E) Fluconazole 150 mg weekly for 6 months

134. A 60-year-old P2 presents to the urogynecology clinic with complaints of urinary


incontinence. She has urinary urgency and can’t make it to the bathroom before leaking a large
amount of urine. She urinate 12 time during the day and gets up two to three times per night to
urinate. She also complaint of urinary leakage during coughing and sneezing. If the urinalysis
revealed normal, what is the next diagnostic plan?
F) Post void residual urine measurement
G) Pad test
H) Voiding Diary
I) Pelvic floor ultrasound
E) Urodynamic study

135. Your patient is a 13-year-old adolescent girl who presents with cyclic pelvic pain. She has
never had a menstrual cycle. She is afebrile and her vital signs are stable. On physical
examination, she has age-appropriate breast and pubic hair development and normal external
genitalia. there is no bulge in the introitus of vagina and no hymenal structure seen. You obtain a
transabdominal ultrasound, which reveals a hematocolpos and hematometra. What is the most
likely diagnosis?

A) Distal Transverse vaginal septum

B) Longitudinal vaginal septum

C) Imperforate hymen

D) Distal Vaginal agenesis

E) MRKH

136. A 89-year-old female patient with multiple, serious medical comorbidities presents to
discuss options for treatment of her high-grade prolapse. The prolapse is externalized and
becoming ulcerated from friction against her undergarments. Her main priority is to “fix or get
rid of this thing,” but her primary care provider has cautioned against a lengthy or open
abdominal procedure. She is not interested in future intercourse. What can you offer this
patient?

A) Total colpocleisis

B) Partial colpocleisis

C) Sacrospinous hysteropexy
D) Ring pessary.

E) Manchester Fothergyl operation

137. A 38 years old multi gravid woman complains of the painless loss of urine, beginning
immediately with coughing, laughing, lifting, or straining. cessation of the activity do not stops
the urine loss. She also complains of frequency with small-voided volume of urine, but no
urgency. This history is most suggestive of ?

A) Fistula

B) Stress incontinence

C) Urge incontinence

D) Overflow incontinence

E) Mixed incontinence

Obstetri 50 soal
01Obs A 30-year-old G3P2002 presents to the obstetrician’s office at 34 weeks for a routine prenatal
visit. She has a history of two prior cesarean sections (low-transverse). The first cesarean section was
performed secondary to fetal malpresentation (footling breech). The patient then had an elective
repeat cesarean section for her second pregnancy. This pregnancy, the patient has had an
uncomplicated prenatal course. The patient is interested in permanent sterilization and wonders if it
would be better to undergo another scheduled cesarean section so she can have a bilateral tubal
ligation performed at the same time. Which of the following statements is true and should be
relayed to the patient?

A) A history of two previous cesarean sections is a contraindication to vaginal birth after cesarean
section (VBAC)

B) Her risk of uterine rupture with attempted VBAC after two prior cesarean sections is 5%

C) Her chance of having a successful VBAC is less than 70%

D) The patient should schedule an elective induction if not delivered by 40 weeks

E) If the patient desires a bilateral tubal ligation, it is safer for her to undergo a vaginal delivery
followed by a postpartum tubal ligation rather than an elective repeat cesarean section with
intrapartum bilateral tubal ligation

02.Obs A 29-year-old G4P3003 with two prior C-sections is diagnosed with a placenta accreta at 28
weeks during follow-up ultrasound of a low anterior placental location. When is placenta accreta
most likely to cause bleeding?

A) during the first stage of labor

B) prior to labor

C) because of consumption coagulopathy


D) after amniotic membrane rupture

E) during attempts to remove it

03Obs A 24-year-old woman (gravida 2, para 0, abortus 1) is seen in the emergency department
because of vaginal bleeding and abdominal cramps. Her LMP was 10 weeks ago. History is unrevealing
except for an induced abortion 2 years ago without complications. She presently denies
instrumentation for abortion. Physical examination reveals a BP of 110/70 mm Hg, pulse 120, and
temperature 101.8°F. The abdomen is tender with slight rebound in the lower quadrants. The pelvic
examination reveals blood in the vault and a foul-smelling discharge from the cervix, which is dilated
to 2 cm. The uterus is 8- to 10-week size and tender, and no adnexal masses are palpated. What is the
most likely diagnosis?

A) choriocarcinoma
B) hydatidiform mole
C) pelvic inflammatory disease (PID)
D) septic abortion
E) twisted ovarian cyst

04Obs A 35-year-old woman (gravida 7, para 5, abortus 1) is in the active phase of labor with the
vertex at –1 station. She complains of abdominal pain with the contractions. At the height of one
contraction, the pain becomes very intense. Following this intense pain, uterine contractions cease.
The maternal systolic BP drops 15 mm Hg. What is the best course of action?

A) immediately perform a pelvic examination

B) place the patient on her side and reassure her

C) manage expectantly

D) begin oxytocin

E) perform an ultrasound

05Obs A 19-year-old G1P0 patient complains of spotting and right-side pain. She had a positive urine
pregnancy test 3 weeks ago. Ultrasound does not identify an intrauterine pregnancy. On laparoscopy
125 cc of blood is seen in the pelvis. There is minimal blood from the tube and a small bit of tissue is
recovered floating free in the peritoneal cavity. This pregnancy is likely which of the following?

A) spontaneous abortion

B) delivery

C) tubal abortion

D) decidual cast

E) Arias-Stella phenomenon
06Obs You are doing postpartum rounds on a 22-year-old G1P1, who vaginally delivered an male
infant at 36 weeks following an induction due to severe preeclampsia. During her labor she
required hydralazine to control her blood pressures. She is on magnesium sulfate for seizure
prophylaxis. Her vital signs are: blood pressure 154/98 mm Hg, pulse 93 beats per minute,
respiratory rate 24 breaths per minute, and temperature 37.3°C. She has adequate urine output
at greater than 40 cc/h. On examination, she is oriented to time and place, but she is somnolent
and her speech is lurred. She has good movement and strength of her extremities, but her deep
tendon reflexes are absent. Which of the following is the most likely cause of her symptoms?

A) Adverse reaction to hydralazine


B) Hypertensive stroke
C) Magnesium toxicity
D) Sinus venous thrombosis
E) Transient i schematic attack

07Obs A 28-year-old G1 at 26 weeks present for her scheduled obstetric appointment. You ordered
OGTT examination that shows fasting blood glucose 102 mg/dL and 2 hours after 75 g oral glucose
185 mg/dL. Her gestational weight gain during pregnancy is 12 kg. Her BMI before pregnancy was 26
kg/m2. What is the appropriate next step in the management of this patient?

A) Schedule nonstress test (NSTs)

B) Schedule fetal growth ultrasound

C) Admit to hospital for fetal monitoring

D) Advise insulin in order to lowering blood glucose

E) Give metformin 3x500mg orally

08Obs A 30-year-old G1 at 26 weeks present for her scheduled obstetric appointment. She has been
mar-ried for 4 years, and had oligomenorrhea. You ordered OGTT examination that shows fasting
blood glucose 98 mg/dL, after 75 g oral glucose 1 hour 175 mg/dL, and 2 hours 167 mg/dL. Her
gestational weight gain during pregnancy is 12 kg. Her BMI before pregnancy was 27 kg/m2. What is
the appropriate next step in the management of this patient?

A) Exercise 120 minutes/week

B) Modification nutrition intake

C) Give metformin 3x500mg orally

D) Ultrasonographic biophysical profile

E) Advise insulin in order to lowering blood glucose


09Obs A 20-year-old primigravida presents at 39 weeks. She has been healthy up to this point. She
has a headache and a loss of appetite. Her face and hands are swollen, and she cannot wear her
rings. Her BP is 168/90 mm Hg, and she has 1+ protein. The fetus has a reassuring monitoring strip.
Which of the following is the best treatment for her preeclampsia?

A) magnesium sulfate
B) delivery either by cesarean or by vaginal
C) an antihypertensive drug that does not affect uterine blood flow
D) gentle diuresis, with careful monitoring of intake and output
E) modified bed rest

10Obs A healthy 20-year-old G1P0 presents for her first OB visit at 10 weeks gestational age. She
denies any significant medical history both personally and in her family. Which of the following
tests is not part of the recommended first trimester blood testing for this patient?

A) Complete blood count (CBC)


B) Screening for human immunodeficiency virus (HIV)
C) Hepatitis B surface antigen
D) Blood type and screen
E) One-hour glucose challenge testing

11Obs A 36 years of G2P1 presents to the antenatal clinic in tertiary referral hospital. She had an
emergency caesarean section for sudden onset hypertension and placental abruption at 30 weeks in
her previous pregnancy. She is currently 12 weeks of gestation and enquiries about further plan of
fetal monitoring in this pregnancy. What is the most accurate advice to do at 12 weeks?

A) BMI calculation

B) Check sFLT-1/PLGF ratio

C) Check urine microalbuminuria

D) Mean arterial pressure calculation

E) Pulsality index uterine artery doppler

12Obs A 32-year-old gravida 3 Para 2, 36 weeks of gestation was transferred from a midwifery-led
unit for high blood pressure. Her blood pressure was 170/100 mmHg, PR 90 bpm, RR 24 x/minute.
Abdominal examination showed FUT 30 cm, ireguler contraction, fetal heart beats 154 bpm. Vaginal
examination showed soft cervix, mid-position, 2 cm dilated, 50% effaced, -2 station . An ultrasound
shows the estimated fetal weight 2600 g, AFI 12 cm, no fetal morphology abnormalities, SDAU 2.9.
She has experienced twice vaginal delivery, baby’s weighed were 3100 g and 3300 g for first and
second baby. On admission, her observations are normal and the cardiotocography (CTG) was
reassuring. You decided to give her MgSO4 and antihypertension with nifedipine for preeclampsia
with severe feature management, and also induction of labor with Oxytocin 5 IU. Adequate
contraction was achieved on 8 mIU/minute. Five hours observation, she complaint rup-ture of
membrane. Vaginal examination showed clear amniotic fluid came out from vagina, cervix dilated 6
cm, 0 station. CTG has become suspicious (baseline 156 bpm, variability 3-5 bpm, no acceleration, no
deceleration, contraction 4x/10’/30”) . What is the most appropriate cause of CTG findings?
A) Cord compression

B) Head compression

C) Magnesium sulphate

D) Placental insufficiency

E) Hyperstimulation of uterus

13Obs A 29-year-old G3P2A0 presents to the emergency center with complaints of abdominal
discomfort for 2 weeks. Her vital signs are BP 120/70mmHg, pulse 90 beats/min, temperature 36 C,
respiratory rate 18 breath/minute. A pregnancy test was positive and an ultrasound of the abdomen
and pelvis reveals a visible 16 weeks gestation located behind a normal appearing 10x6x5.5 cm
uterus. Both ovaries appear normal. No free fluid is noted. Which of the following is the most likely
cause of these findings?

A) Ectopic ovarian tissue

B) Fistula between the peritoneum and the uterine cavity

C) Primary peritoneal implantation of the fertilized ovum

D) Tubal abortion

E) Uterine rupture of prior cesarean section scar

14Obs Mrs. A, 24-year-old came to the ER with complaints of headaches since the last day of
examinations obtained expecting her first child, gestational age 32 weeks with blurred vision and
denied heartburn. On physical examination found BP 190/120 mmHg, pulse 90 x/m, breathing 16
x/m. Leopold found the lower left back head, FHR 140 bpm, irregular contraction. Pelvic score of 1
was found, pelvis size wide. Laboratory investigation; hemoglobin 11.5 g%, platelets 9000/ mm3,
LDH 510 iu/L, Proteinuria +2, ALT 10 u/L, AST 15 u/L What is the best diagnosis for Mrs. A?

A) HELLP syndrome

B) Severe preeclampsia

C) Chronic hypertension

D) Gestational hypertension

E) Superimposed preeclampsia

15Obs Which of the following is true regarding COVID-19 in pregnancy?

A) No anti-viral was approved by FDA for pregnant women


B) Preterm delivery and preeclampsia are increased in pregnant women with COVID-19

C) Vertical transmission is unlikely because placenta has low expression of ACE2 receptor

D) Recommendation mode of delivery in pregnant women with COVID-19 is normal delivery with 1
health provider

E) Evidence showed that pregnancy offers an altered immunity scenario which may allow severe
COVID-19 disease

16Obs Mrs. D 39 years old G4 P3 34 weeks of gestation arrives at delivery ward with severe
dyspnoe. She looks really anxious, Her vital signs show: BP 180/110mmHg; PR 110x/min; RR
26x/min; 36.4oC, SpO2 95%. Conjunctiva not pale. Heart: normal heart sound, no murmur or
gallop.Lung: Vesiculer with rales and no wheezing.Fundal height 30cm, head presentation, FHR
170bpm, no contractionLaboratory findings: CBC Hb 10;Ht 32;L 12,000; Platelet 120,000What is the
mechanism that can induce this condition?

A) Right heart failure

B) Community acquired pneumonia

C) High oncotic pressure

D) Diastolic dysfunction

E) Immune response

17Obs A 39-year-old woman (G4P4) has had a complicated pregnancy with Graves' disease. She
has delivered 4 hours ago. Her thyroid levels and TSH levels have been in the normal range
throughout gestation. Among which of the following will her infant most likely be?

A) hypothyroid infant
B) mongoloid infant
C) hyperthyroid infant
D) Infertile infant
E) infant with ambiguous genitalia

20Obs A 27 year old, G1 40 weeks gestation, was referred by midwives due to labor dystocia. Based
on partograph, cervical dilatation of 4 cm was happened at 08.00 am. At 12.00 pm, cervical
dilatation remains the same, with contraction 2x/10’/20” and intact membrane. Estimated fetal
weight is 3000 gram with adequate pelvic examination. This condition can be classified as?

A) Protraction disorder

B) Incoordinate uterine action


C) Arrest disorder

D) Prolonged of latent phase

E) Prolonged of active phase

21Obs A 17 year old, G1 20 weeks gestation, come to your outpatient clinic for routine antenatal
care. No complaint regarding pregnancy. During sonographic examination, you find ventriculomegaly
appearance as seen below.[gambar 21]

Severe ventriculomegaly is diagnosed when lateral ventricular atrial width exceed what threshold ?

A) 5 mm

B) 7 mm

C) 10 mm

D) 15 mm

E) 19 mm

23Obs You are counselling a couple in your clinic who desire VBAC (Vaginal Birth After Cesarean
Section). Her baby is in a vertex presentation, appropriate size for 37 weeks, and her previous
low transverse procedure was for breech presentation. In providing informed consent, in which
of the following ways do you explain the risk of uterine rupture?

A) Less than 1%
B) Between 2% and 5%
C) 15–20%
D) Dependent on the length of her labor
E) Dependent on the location and proximity of the scar site to the placental implantation
24Obs A 33-year-old woman come to outpatient Clinic with recurrent pregnancy loss. Patient has
been married for 5 years, has already examined for infertility workup with normal results. Patient
has experienced 3 times abortion. The first and the second pregnancy was in 6 weeks gestation, the
third pregnancy was in 8 weeks gestation. Patient had never performed diagnostic work up
regarding this condition. What cells contribute in the early implementation process of pregnancy?

A) Basofil

B) Eosinofil

C) Antiphosfolipid antibodies

D) Natural killer cell

E) T cells

25Obs A 39-year-old G2P1 presents to your outpatient clinic with pregnancy test was positive. She
has history of 10 weeks amenorrhea. Her first child has Down Syndrome. She is asking regarding
aneuploidy screening. Which of the following is the most appropriate regarding aneuploidy
screening?

A) Risk of miscarriage is greater in amniocentesis than chorionic villus sampling


B) cfDNA testing as a diagnostic test
C) cfDNA has a high sensitivity and specificity for trisomy 18 and trisomy 21
D) If a fetal structural anomaly is identified on ultra-sound examination, diagnostic testing
cfDNA should be offered
E) The fetal fraction, the amount of the cell-free DNA in the maternal blood that is of fetal
origin, at least 4% is required for accurate test results.

26Obs Which of the following factors are least likely to be linked with higher first-Trimester
miscarriage rates?

A) Maternal age > 40 years old

B) Parvovirus infection

C) Diabetes mellitus

D) Obesity

E) Thyroid disorders

27Obs At outpatient clinic, you are now examining a 38 week-pregnant patient with a BMI of
15. She denies history of chronic diseases but she admits that she has lost her appetite since the
first trimester. Her vital signs are within normal limit. Her fundal height is only at the level of 2
fingers above her umbilicus and the fetus is easily palpated. The CTG reveals poor variability. Which
of the following is true regarding maternal nutrition during pregnancy?

A) Providing macronutrient supplementation to undernourished women consistently lowers rates of


newborns.

B) Providing micronutrient supplementation to undernourished women consistently lowers rates of


newborns.

C) For all maternal weight categories, excessive maternal weight gain during pregnancy is associated
with newborns.

D) For all maternal weight categories, maternal weight gain in the first trimesters that is less than
recommended is associated with restriction.

E) For all maternal weight categories, maternal weight gain in the second and third trimesters that is
less than recommended is associated with restriction.

28Obs At outpatient clinic, you are now examining a 38 week-pregnant patient with a BMI of
15. She denies history of chronic diseases but she admits that she has lost her appetite since the
first trimester. Her vital signs are within normal limit. Her fundal height is only at the level of 2
fingers above her umbilicus and the fetus is easily palpated. The CTG reveals poor variability. During
the ultrasonography examination, the findings that will be found related to her conditions are

A) Normal AFI and S/D ratio > 3.

B) Normal AFI and S/D ratio < 3.

C) Oligohydramnions and S/D ratio < 3.

D) Oligohydramnions and S/D ratio0 > 3.

E) Both normal AFI and oligrohydramnions are possible, and S/D ratio < 3.

29Obs Mrs. 37 years old, G3P2, 35 wga, came to ER referred from nearby PHC due to high blood
pressure, on admission her blood pressure was 170/110 mmHg. She complained of epigastric pain
followed by nausea and vomitting. She denied having contraction, water broke, and bloody show,
from her physical examination you found her fundal height was 3 fingers above navel, head
presentation, there was no contraction and no dilation of cervix. You decided to terminate the
pregnancy by emergency cesarean section due to impending eclampsia. The baby born with Apgar
Score 6/8, bodyweight 1480 g, and diminished amniotic fluid. When you were performing informed
consent before the cesarean section, you were explaining the neonatal complication of IUGR, one of
them is ?

A) Delayed lung maturation

B) Cerebral Palsy
C) Retinopathy

D) Susceptibility to infection

E) Transient tachypneu of the newborn

30Obs Mrs. 37 years old, G3P2, 35 wga, came to ER referred from nearby PHC due to high blood
pressure, on admission her blood pressure was 170/110 mmHg. She complained of epigastric pain
followed by nausea and vomitting. She denied having contraction, water broke, and bloody show,
from her physical examination you found her fundal height was 3 fingers above navel, head
presentation, there was no contraction and no dilation of cervix. You decided to terminate the
pregnancy by emergency cesarean section due to impending eclampsia. The baby born with Apgar
Score 6/8, bodyweight 1480 g, and diminished amniotic fluid. You suspected placental insufficiency
due to high blood pressure as the risk factor of IUGR, to confirm the diagnosis, the macroscopic
appearance of the placenta and umbilical cord should be ?

A) Placental weight less than 1/7 babyweight

B) Thick wharton jelly

C) Maternal side with wide hematome

D) Umbilical cord with single umbilical artery

E) Umbilical cord length < 50 cm

32Obs A 36 years old patient, G4P3, 37wga, obese patient came into policlinic with diagnosis
diabetes mellitus in pregnancy and history of previous CS 2x. She is scheduled for elective cesarean
section at 38 completed weeks of gestational age. She had a poor glycemic control with oral
antidiabetic regiments. 6 hours prior to her surgery you found that her random blood glucose is
297g/dL. What do you think is the best method for her pre-operative glycemic control?

A) Sliding scale using insulin

B) Increase the dose of her oral antidiabetic medication

C) Combine her oral antidiabetic with another type of oral antidiabetic

D) Combine her oral antidiabetic with long acting insulin

E) Postpone her surgery due to uncontrolled blood glucose level


33Obs During an operation, a midline incision was made at an anatomic location 2 cm below the
umbilicus. Which of the following lists (in order) the layers of the anterior abdominal wall as they
would be incised or separated?

A) Skin, subcutaneous fat, superficial fascia (Camper’s), deep fascia (Scarpa’s), fascial muscle cover
(anterior rectus sheath), rectus muscle, a deep fascial muscle cover (posterior rectus sheath),
preperitoneal fat, and peritoneum

B) Skin, subcutaneous fat, superficial fascia (Scarpa’s), deep fascia (Camper’s), fascial muscle
covering (anterior abdominal sheath), transverse abdominal muscle, a deep fascial muscle cover
(posterior rectus sheath), preperitoneal fat, and peritoneum

C) Skin, subcutaneous fat, superficial fascia (Camper’s), deep fascia (Scarpa’s), fascial muscle cover
(anterior rectus sheath), rectus muscle, a deep fascial muscle cover (posterior rectus sheath),
peritoneum, and preperitoneal fat

D) Skin, subcutaneous fat, superficial fascia (Scarpa’s), deep fascia (Camper’s), fascial muscle cover
(anterior rectus sheath), rectus muscle, a deep fascial muscle cover (posterior rectus sheath),
preperitoneal fat, and peritoneum

E) Skin, subcutaneous fat, superficial fascia (Camper’s), deep fascia (Scarpa’s), fascial muscle cover
(anterior rectus sheath), transverse abdominal muscle, a deep fascial muscle covering (posterior
rectus sheath), preperitoneal fat, and peritoneum

34Obs Numerous physiologic changes develop over the course of pregnancy and postpartum,
however the greatest impact on potentially compromised cardiovascular system is?

A) Stroke volume decrease after delivery of placenta

B) Increased total plasma volume in second trimester

C) Maximum heart rate increases in the second trimester

D) Decreased SVR in the first trimester, and increase after 32 weeks

E) Maximal increase in cardiac output occurs during second stage of labor

35Obs Which one of the following gives the highest score in Placenta Acreta Index ? What is the
most potential perioperative problem of this patient?

A) Lacunae Grade 3

B) Bridging Vessels

C) > 2 Caesarean deliveries

D) Anterior Placenta Previa

E) Sagital Smallest Myometrial Thickness 1mm


36Obs A 29-year-old G1P0 patient at 15 weeks gestational age presents to your office complaining of
some shortness of breath that is more intense with exertion. She has no significant past medical
history and is not on any medication. The patient denies any chest pain but sometimes feels as
though her heart is pounding. She is concerned because she has always been very athletic and
cannot maintain the same degree of exercise that she was accustomed to prior to becoming
pregnant. On physical exam, her pulse is 90/min. Her blood pressure is 90/50 mmHg. On cardiac
exam, a systolic ejection murmur is identified. The lungs are clear to auscultation and
percussion. What would you explain to the patient above?

A) Cardiac output is increased 1.5 L/min


B) Blood flow to uterus is increase by 200 mL/min
C) Pulmonary blood pressure is increased
D) Pulse pressure is decreased
E) Stroke volume remains unchanged during pregnancy

37Obs A 29-year-old G1P0 patient at 15 weeks gestational age presents to your office complaining of
some shortness of breath that is more intense with exertion. She has no significant past medical
history and is not on any medication. The patient denies any chest pain but sometimes feels as
though her heart is pounding. She is concerned because she has always been very athletic and
cannot maintain the same degree of exercise that she was accustomed to prior to becoming
pregnant. On physical exam, her pulse is 90/min. Her blood pressure is 90/50 mmHg. On cardiac
exam, a systolic ejection murmur is identified. The lungs are clear to auscultation and
percussion. What is the most possible cause of her complain?

A) Myocardial infarction

B) Valvular heart disease

C) Pulmonary embolism

D) Eisenmenger syndrome

E) Normal cardiovascular changes

38Obs Which of the following is the best term to diagnose maternal primary CMV infection?

A) Ig M negative, Ig G positive
B) Ig M negative, Ig G negative
C) Ig M positive, Ig G negative
D) Ig M positive, Ig G positive high avidity
E) Ig M positive, Ig G positive low avidity

39Obs A previously energetic woman complains of crying, loss of appetite, difficulty in sleeping, and
feeling of low self-worth, beginning approximately 3 days after a normal vaginal delivery. These
feelings persisted for approximately 1 week and then progressively diminished. Pregnancy could
exacerbates some coexisting mental disorders due to ?
A) Pregnancy-related shifts in monoamine neurotransmitter levels

B) Body changes cannot easily accepted by women

C) Lack of affection from spouse and family members

D) Lack of certain minerals in pregnancy that could affect limbic system

E) Certain medication could interfere brain function during pregnancy

40Obs A previously energetic woman complains of crying, loss of appetite, difficulty in sleeping, and
feeling of low self-worth, beginning approximately 3 days after a normal vaginal delivery. These
feelings persisted for approximately 1 week and then progressively diminished. Patient with
psyciathric disorders can result to poor pregnancy outcome such as ?

A) Preeclampsia

B) Major congenital anomaly

C) Increased rate of cesarean section

D) Increased risk of post partum hemorrhage

E) Low birthweight

41Obs A 29 y.o G3P2 at 35 weeks of gestation came to your outpatient clinic. She is complained
decreased fetal movement. Her previous pregnancy went uncomplicated, ended in full term with
normal spontaneous vaginal delivery. You found an indication for early delivery, but you concern of
his lung maturity. Regarding fetal lung maturity, which of the following is true ?

A) Type I pneumocytes secrete surfactant

B) Typically, lecithin decreases as the lung matures

C) Sphingomyelin decreases beyond 24 weeks

D) A low L/S ratio is associated with fewer cases of respiratory distress syndrome (RDS)

E) A lecithin to sphingomyelin (L/S) ratio greater than 2 is indicated for early delivery

42Obs Which of the following is the most common ultrasound findings in CMV infection ?

A) Fetal hydrops

B) Hepatomegaly

C) Placentomegaly
D) Ventriculomegaly

E) Intrauterine Growth Restriction

44Obs Concerning middle cerebral artery Doppler velocimetry, which of the following is true?

A) It is useful for detection and management of fetal anemia of any cause

B) It is superior to the modified biophysical profile in forecasting pregnancy outcomes

C) It is useful to detect fetal growth restriction

D) In those with brain sparing, decreased blood flow from reduced cerebrovascular impedance is
detected

E) It was found to be inferior to amniocentesis and amnionic fluid spectral analysis for predicting
fetal anemia

45Obs A 23 year-old G1 32 weeks is being admitted to the hospital because of preterm contraction.
The patient complaint regular contraction. Antenatal care was done regularly in PHC. No remarkable
abnormality was found during ANC. BMI before pregnancy was 30 kg/m2, weight gain during
pregnancy is 14 kg. Abdominal examination showed FUT 36 cm, regular contractions, fetal heart
beats 154 bpm. Speculum examination showed closed ostium uteri externa. An ultrasound shows
the estimated fetal weight 2400 g, AFI 30 cm, no fetal morphology abnormalities, placenta
implanted in anterior corpus, cervical length 1.8 cm, funneling positive. Laboratory results were Hb
10,7 g/dL, Ht 33%, Leucocyte 13.500, Thrombocyte 315.000 MCV 82 MCH 30. What is the most
appropriate next step in the management of this patient?

A) Give intravenous iron

B) Schedule for OGTT test

C) Give antibiotic prophylaxis

D) Schedule for cervical cerclage

E) Give MgSO4 for neuroprotection

46Obs A 18-year-old G1 at 32 4/7 weeks presents for her scheduled obstetric (OB) appointment.
Ultrasound charts showed below.
The patient denies any complaints today. Fetal movement is active. BMI before pregnancy was 19.6
kg/m2, gestational weight gain is 7 kg. Vital sign are normal. Abdominal examination today shows a
gravid uterus measuring 29 cm. Fetal heart tones (FHTs) are in the 140s. What is the most
appropriate diagnosis of this patient?

A) Underweight
B) Small for gestational age
C) Appropriate for gestational age
D) Symmetrical intrauterine growth restriction
E) Asymmetrical intrauterine growth restriction

47Obs A 34-year-old G1 at 26 4/7 weeks presents for her scheduled obstetric (OB) appointment. The
patient denies any complaints today. Fetal movement is active. BMI before pregnancy was 29.6
kg/m2, gestational weight gain is 9 kg. Vital sign are normal. Abdominal examination today shows a
gravid uterus measuring 26 cm. Fetal heart tones (FHTs) are in the 140s. Laboratory results were Hb
10,1 g/dL, Ht 30%, Leucocyte 10.500, Thrombocyte 165.000, MCV 78, MCH 28. OGTT result was
fasting 102 mg/dL and 2 hours after 75 glucose 156 mg/dL. What is the most likely diagnosis for this
case?

A) Maternal overweight
B) Iron deficiency anemia
C) Gestational diabetes mellitus
D) Pre-gestational diabetes mellitus
E) Inadequate maternal weight gain

48. A 35-year-old woman, G3P2, presents to labor and delivery (L&D) suite at 33-week gestation
referred by a midwife with BP 180/120 mmHg. BP on arrival is 170/105 mmHg. Urine protein is
negative on dipstick. The patient has no history of high blood pressure before pregnancy and denies
any complaints today. What is the most likely diagnosis of the patient ?
A) Preeclampsia

B) Chronic hypertension

C) Gestational hypertension

D) Superimposed preeclampsia

E) Preeclampsia with severe feature

49. A 35-year-old woman, G3P2, presents to labor and delivery (L&D) suite at 33-week gestation
referred by a midwife with BP 180/120 mmHg. BP on arrival is 170/105 mmHg. Urine protein is
negative on dipstick. The patient has no history of high blood pressure before pregnancy and denies
any complaints today. What is the appropriate next step in the management of this patient?

A) Schedule for a biophysical profile (BPP)


B) Schedule for fetal growth ultrasound next week
C) Admit patient to the hospital for lung maturation
D) Admit patient to hospital for termination pregnancy
E) Ask patient to increase nutrient intake, particularly protein intake

50. A 35-year-old woman, G3P2, presents to labor and delivery (L&D) suite at 33-week gestation
referred by a midwife with BP 180/120 mmHg. BP on arrival is 170/105 mmHg. Urine protein is
negative on dipstick. The patient no history of high blood pressure before pregnancy and denies any
complaints today. What is the appropriate maternal weight gain of this case (based on ASIAN BMI
category criteria) ?

A) < 9 kg
B) 5 – 9 kg
C) 7 – 11.5 kg
D) 11.5 – 16 kg
E) 12.5 – 18 kg

You might also like